9/3-MBE EVIDENCE

Ace your homework & exams now with Quizwiz!

Question 6483 A woman is on trial for a burglary that took place at about six in the evening on November 1. A surveillance video from a local gas station shows that the woman visited the gas station at seven in the evening on November 1. The woman alleges as an alibi that she was actually at the gas station at six in the evening. She argues that the gas station failed to change the time display on its camera to reflect the end of Daylight Savings Time on the morning of November 1. The judge, on his own initiative and after first giving the prosecutor the opportunity to object, took judicial notice of the fact that Daylight Savings Time did end in the year in question on November 1. The judge instructed the jury that it may or may not accept any judicially noticed fact as conclusive. Were the judge's actions with regard to judicial notice of this fact proper? Answers: Yes, because the court may take judicial notice on its own initiative. Yes, because the court gave the prosecution an opportunity to be heard on the propriety of taking judicial notice before doing so. No, because the fact is not one that is generally known within the territorial jurisdiction of the trial court. No, because the court should have instructed the jury that it is required to accept the noticed fact as conclusive.

Answer choice A is correct. A court may take judicial notice at any time during a proceeding, including on appeal, whether upon request of a party or by the court's own initiative. Answer choice B is incorrect. When a party makes a timely request, the judge must give the party an opportunity to be heard on the propriety of taking judicial notice and the nature of the fact to be noticed. However, a judge is not required to provide this opportunity before taking judicial notice of an adjudicative fact. Answer choice C is incorrect. Although judicial notice may be taken of an adjudicative fact that is not subject to reasonable dispute because it is generally known within the territorial jurisdiction of the trial court, judicial notice may also be taken of an adjudicative fact that can be accurately and readily determined from sources whose accuracy cannot reasonably be questioned. Here, the date on which Daylight Savings Time ends is such a fact. Answer choice D is incorrect because this is a criminal case, and in a criminal case, the jury must be instructed that it may or may not accept any judicially noticed fact as conclusive, as the judge properly did here.

Question 7530 The driver of a car was involved in an accident with a bicyclist. The bicyclist died as a result of his injuries, and the state has brought a vehicular manslaughter case against the driver, alleging that the driver caused the accident when he failed to stop at a red light. At trial, the driver calls his friend, who was not present at the scene of the accident, as a witness. The friend plans to testify that he knows that the driver is a safe driver, and that it is unlikely that the driver failed to stop at a red light on the day of the accident. The prosecution objects, seeking to exclude the friend's testimony. Which of the following is the best basis for the prosecution's objection? Answers: The friend lacks personal knowledge of the accident. The friend is not an expert on the issue of traffic accidents. The friend's testimony offers character evidence in a criminal case. The friend is biased in favor of the driver.

Answer choice A is correct. A non-expert witness must have personal knowledge of a matter in order to testify about that matter. Personal knowledge may be established by the witness's own testimony as well as through other means. In this case, the friend intends to testify as to what he believes is likely to have occurred on the day of the accident, but the friend does not have any personal knowledge of the accident. Answer choice B is incorrect because nothing indicates that the friend is being offered as an expert on traffic accidents. Additionally, even lay witnesses may testify as to their own opinion if the opinion (i) is rationally based on the witness's perceptions and (ii) would be helpful to a clear understanding of the witness's testimony or the determination of a fact in issue. Therefore, this is not the strongest objection to the friend's testimony. Answer choice C is incorrect. In a criminal case, a defendant is permitted to introduce evidence of his good character as being inconsistent with the type of crime charged. Proof of good character offered by the defendant must be in the form of reputation testimony or opinion testimony. Here, the defendant is permitted to offer evidence of his good character, so an objection as to the use of character evidence in general would be improper. However, the friend's testimony regarding the defendant's character is not in the proper form—it is not his opinion or the defendant's reputation in the community. Therefore, it cannot be admitted here. Answer choice D is incorrect because bias is not a basis for excluding testimony by a witness. Rather, the prosecution would impeach the witness during cross-examination by showing the friend is biased toward the driver because of their friendship.

Question 7187 A plaintiff and a defendant were involved in a car accident in an intersection. Both parties sustained minor injuries in the accident, and the plaintiff subsequently brought an action for negligence against the defendant. Although there was evidence that both parties may have been negligent, the plaintiff sought to establish that the defendant's vehicle did not slow down when it entered the intersection. The plaintiff's attorney called a passenger who was in the defendant's vehicle at the time of the accident. The passenger testified that when she and the defendant got out of the car after the accident, the defendant told her, "It felt like the brakes failed, but that's not possible, my mom just had the car serviced." The defendant objected to the admission of this testimony. Should the court sustain the defendant's objection to the passenger's testimony? Answers: No, because the testimony can be offered by the plaintiff as a statement of a party opponent. No, because the testimony is only admissible for impeachment purposes. Yes, because the defendant's statement to the passenger was not based on personal knowledge. Yes, because the defendant's statement was not against his interest at the time it was made.

Answer choice A is correct. A statement made by a party to the current litigation is not hearsay if it is offered by an opposing party. Unlike with the statement against interest hearsay exception, an opposing party's statement need not have been against the party's interest at the time that it was made. Therefore, even if this statement was interpreted as being in the defendant's interest because it tends to establish that the car had been recently serviced, it will still be admissible as an opposing party's statement. Answer choice B is incorrect because the statement by the defendant is not only admissible for impeachment purposes; it can also be admitted as substantive evidence as an opposing party's statement. Answer choice C is incorrect because an opposing party's statement does not need to be based on personal knowledge to be admissible as nonhearsay. Answer choice D is incorrect because, unlike with the statement against interest hearsay exception, an opposing party's statement need not have been against the party's interest at the time that it was made.

Question 6495 A tenant has sued his landlord alleging that the landlord failed to repair a defective step of a staircase in the common area of the apartment building. The tenant suffered injuries when the top step on a flight of stairs broke and the tenant fell down the stairs. After the accident, the tenant discussed the incident with a neighbor living in the same apartment building. The neighbor said that her last roommate had almost fallen when the same step had cracked under his weight, and that he had told her that he had warned the landlord about the step when he turned in his last rent check. Neither party has attempted to subpoena the former roommate to testify at trial. The landlord has testified that he never saw the defect in the step. Can the neighbor testify at trial as to her former roommate's statement to prove that the stairs were defective? Answers: No, because it is hearsay not within any exception. No, because it is only admissible to prove that the landlord was on notice of a possible defect. Yes, because the declarant is unavailable as a witness. Yes, to impeach the landlord's testimony by contradictory evidence.

Answer choice A is correct. A statement that contains hearsay within hearsay may be admissible as long as each part of the combined statement conforms to a hearsay exception. Although the statement to the landlord may be admissible to show its effect on the recipient if it were offered by the roommate, no hearsay exception allows the neighbor to testify about the statement made to her by the roommate. Answer choice B is incorrect. Although a statement offered to show the effect on the person who heard it is not hearsay, the former roommate's statement to the neighbor about the statement to the landlord is hearsay and there is not an applicable hearsay exception to permit the introduction of this statement into evidence. In other words, if the former roommate were to testify, that testimony would be admissible. Answer choice C is incorrect because a living declarant whom no party has attempted to subpoena is not considered an unavailable declarant. Answer choice D is incorrect. A witness may be impeached by evidence that contradicts the witness's testimony. Here, the statement would not contradict the landlord's assertion that he never saw the defect.

Question 3019 A defendant charged with conspiracy to commit murder requested a meeting with the prosecutor to discuss the defendant's cooperation. At the beginning of this pre-trial meeting, the prosecutor indicated that she would talk with the defendant only if the defendant agreed that statements made by the defendant during the meeting could be used to impeach the defendant's testimony if a plea deal could not be reached and the defendant chose to testify at trial. The defendant, after consulting with his attorney, knowingly and voluntarily agreed. No plea agreement was subsequently reached. At trial, the defendant testified. When the prosecutor attempted to impeach the defendant's testimony with statements that the defendant had made during the pre-trial meeting, the defendant's attorney objected. Should the court permit the prosecutor to ask the defendant about such statements? Answers: Yes, because the defendant knowingly and voluntarily waived his right to have his statements made during a plea negotiation excluded. Yes, because the criminal defendant is a party-opponent whose statements are admissible as non-hearsay. No, because a statement made by a defendant during a plea negotiation is inadmissible against a criminal defendant for any purpose. No, because the prosecution's use of the statements violates the defendant's privilege against self-incrimination.

Answer choice A is correct. Although statements made by a defendant during plea negotiations are generally inadmissible pursuant to Federal Rule 410, the protection afforded by this rule may be waived by the defendant. In this case, the defendant knowingly and voluntarily, and in the presence of counsel, waived this exclusion. Answer choice B is incorrect because, although the defendant's statements do constitute non-hearsay as statements of a party to the action, the prosecution is not offering the statements for substantive purposes (i.e., for their truth) but for impeachment purposes. Moreover, the exclusion for statements made by a defendant during plea negotiations trumps the admission of such statements as non-hearsay even where they are admitted for substantive purposes. Answer choice C is incorrect because, although there is an exclusion for statements made by a defendant during plea negotiations which applies to its use for impeachment as well as substantive purposes, a defendant may knowingly and voluntarily waive the protection. Answer choice D is incorrect because the privilege against self-incrimination, which would prevent the prosecution from using the defendant's compelled statements against him, would not apply to statements given voluntarily.

Question 7500 A defendant was charged with illegal possession of a firearm. At trial, a witness testified that the defendant had shot at him with the firearm and he had returned the defendant's fire. The defendant called a police officer to the stand in order to impeach the witness by testifying that the witness had told her that he, not the defendant, had fired first. Instead, the police officer testified that she did not remember what the witness had told her. The defendant sought to introduce a properly authenticated sworn statement made by the police officer at a prior hearing that the witness told her he had fired first. Can the court refuse to admit this statement into evidence? Answers: Yes, because it involves a collateral matter. Yes, because it is offered for the truth of the matter asserted. No, because it is a prior inconsistent statement. No, because it is admissible as the former testimony of the police officer.

Answer choice A is correct. Although the Federal Rules do not explicitly prohibit impeachment on collateral issues, a court may refuse to admit evidence related to a collateral issue under the Rule 403 balancing test. Generally, a party may not impeach the credibility of a witness by introducing extrinsic evidence of a collateral matter. Instead, the party must accept the witness's testimony. Here, the defendant, by calling the police officer as a witness, sought to introduce evidence to impeach the witness on a collateral matter because whether the witness or the defendant fired first, although relevant if the defendant had been charged with assault, is not relevant to the crime with which the defendant was charged—illegal possession of a firearm. Consequently, the court could refuse to admit extrinsic evidence that would only serve to impeach the original witness because the continued examination of this matter could serve to confuse an issue the jury was to decide—whether the defendant illegally possessed a firearm. Answer choice B is incorrect. The sworn statement was not introduced as evidence that the witness had fired first, but merely to impeach the witness's statement that the defendant had fired first. Moreover, even assuming the sworn statement was introduced for the truth of the matter asserted, the court could refuse to admit it into evidence because it relates to a collateral matter. Answer choice C is incorrect. Because the police officer testified that she did not remember what the witness had told her, her prior statement, although made under oath, does not contradict her current testimony. More importantly, even assuming the sworn statement was admissible as a prior inconsistent statement, the court could refuse to admit it into evidence because it relates to a collateral matter. Answer choice D is incorrect. The sworn statement could be admissible under the hearsay exception for former testimony because the police officer was unavailable as a witness due to her inability to remember what the witness had told her. However, the sworn statement was not introduced as evidence that the witness had fired first, but merely to impeach the witness's statement that the defendant had fired first. Moreover, even assuming the sworn statement was admissible under the former testimony hearsay exception, the court could still refuse to admit it into evidence because it relates to a collateral matter.

Question 3016 The holder of a patent for hybrid corn sued both an unlicensed wholesaler of that product and a retailer who purchased the corn from the wholesaler for alleged patent infringement. The patent holder reached an agreement with the retailer in which the holder released the retailer from liability in exchange for the payment of a nominal amount. After properly authenticating the agreement, the wholesaler sought to introduce it into evidence solely for the purpose of determining damages. The patent holder objected to the introduction of the agreement. Should the court admit the agreement into evidence? Answers: No, because the agreement constituted the acceptance of a settlement offer. No, because the danger of unfair prejudice outweighs the probative value of the agreement. Yes, because the agreement is being introduced solely for the purpose of determining the amount of damages. Yes, because the wholesaler was not a party to the agreement.

Answer choice A is correct. Evidence of a settlement offer, including evidence of the acceptance of such an offer, is not admissible for the purpose of establishing the validity of a claim or the amount of damages. For this reason, answer choice C is incorrect. Answer choice B is incorrect because it fails to correctly state the law. In order for evidence to be inadmissible under Federal Rule 403, the dangers of unfair prejudice must substantially outweigh its probative value. Answer choice D is incorrect because when there are more than two parties, a settlement agreement entered into by a party with an adverse party cannot be used by a remaining adverse party to prove or disprove the amount of an unsettled claim.

Question 3045 A plaintiff filed suit against a defendant supermarket for injuries he sustained when he slipped on a piece of lettuce in the supermarket's produce aisle. A supermarket employee who witnessed the plaintiff's fall prepared a written summary of the events that had occurred in order to alert the supermarket's management. At trial, the supermarket's lawyer called the employee as a witness and, after her testimony, asked her to read her summary to the jury. The plaintiff objected to the testimony. May the court admit the testimony at this time over the plaintiff's objection? Answers: Yes, if the employee is unable to remember the actual events. Yes, because the employee is on the witness stand and can be cross-examined. No, because it is hearsay not within any exception. No, because such testimony may only be offered into evidence by an adverse party.

Answer choice A is correct. FRE 803(5) creates an exception to the hearsay rule for a memorandum or record concerning a matter about which a witness once had knowledge but now has insufficient recollection to enable the witness to testify fully and accurately, shown to have been made or adopted by the witness when the matter was fresh in the witness's memory and to reflect that knowledge correctly. Here, the summary was made immediately after the slip-and-fall, while the witness's memory of the events was fresh. If admitted, the memorandum or record may be read into evidence but may not itself be received as an exhibit unless offered by an adverse party. Here, the written summary is being read into evidence only. Answer choice B is incorrect. The hearsay rule generally applies to any out-of-court statement, even if the declarant is available to be cross-examined on the witness stand. To be admissible, a statement that is hearsay must come within one of the exceptions to the hearsay rule. In this case, the summary meets the past recollection recorded exception. Answer choice C is incorrect because the summary, if the employee is unable to remember the events, meets the requirements of the past recollection recorded exception to the hearsay rule. Answer choice D is incorrect because, as a past recollection recorded, the summary may be read into evidence. The rule only requires the offering by an adverse party if the summary itself is being offered as an exhibit.

Question 6967 A woman was prosecuted for burglary after she broke into a man's home one night while he was out of town and stole a diamond ring. The woman claimed that the man was her fiancé, that she had forgotten her engagement ring at his house, and that she was afraid it would be stolen while he was away. At trial, it was established that the man was not the woman's fiancé, that they had never met, and that the man bought the engagement ring to propose to his girlfriend, the woman's co-worker. The defense called a psychiatrist to the stand to give his expert opinion about the woman's state of mind at the time of the burglary. The psychiatrist testified that after conducting a thorough psychological examination of the woman, his conclusion was that the woman lacked the requisite intent to commit a felony inside of the man's home. Is the psychiatrist's testimony admissible? Answers: No, because the psychiatrist is opining about whether the woman had the specific intent to commit a felony when she broke into the man's house. No, because the psychiatrist stated his opinion about the woman's state of mind before testifying about the underlying facts and data that supported it. Yes, because the psychiatrist's opinion would help the jury to determine whether the woman had the intent to commit a felony when she broke into the man's house. Yes, because the use of a psychological examination is a reliable method of determining a person's state of mind.

Answer choice A is correct. Generally, an expert's opinion may be admissible even though the opinion embraces an ultimate issue in the case, including the defendant's state of mind. However, an expert may not state an opinion about whether a criminal defendant had the requisite mental state of any element of the crime charged or of a defense. That determination lies in the province of the trier of fact. Here, the psychiatrist stated that the woman lacked the specific intent to commit a felony inside of the man's home, which is an element of the crime of burglary. Therefore, the psychiatrist's testimony is inadmissible. Answer choice B is incorrect because an expert may state an opinion and give the reasons for the opinion without first testifying as to the underlying facts or data. Answer choice C is incorrect because an expert may not state an opinion about whether a criminal defendant had the requisite mental state of any element of the crime charged or of a defense, regardless of how helpful that opinion would be to the jury. Answer choice D is incorrect. Even if a psychological examination is a reliable method of determining a person's state of mind, the issue here is that the psychiatrist testified as to whether the woman had the specific intent to commit a felony inside of the man's home, and he is not permitted to do so.

Question 7502 The driver and owner of an SUV filed a complaint against the driver of a sports car for damages stemming from an accident that occurred one evening when it was lightly raining. The driver of the sports car filed a counterclaim against the driver of the SUV. At trial, the driver of the sports car called a passenger in the SUV, who is a friend of the driver of the SUV, to testify. The passenger, who was not hurt in the accident, intends to testify that she told a friend the day after the accident that the driver of the SUV should have had his windshield wipers on when the accident occurred. Is this testimony admissible? Answers: No, because it is hearsay. No, because the statement is inadmissible opinion testimony. Yes, because the declarant and the testifying witness are the same person. Yes, because it is a declaration against interest.

Answer choice A is correct. Hearsay is a statement that the declarant makes at a time other than while testifying at the current trial or hearing (i.e., an out-of-court statement) that is offered to prove the truth of the matter asserted. Here, the statement is offered by the driver of the sports car to show that the driver of the SUV should have had his windshield wipers on at the time of the accident, i.e., for the truth of the matter asserted. Consequently, it is hearsay and thus inadmissible. Answer choice B is incorrect because, although generally a lay (non-expert) witness is not permitted to testify as to her opinion, lay opinions are admissible with respect to common-sense impressions such as appearance, intoxication, speed of a vehicle, or another's emotions. Here, the question of whether it was raining hard enough for the driver to turn on his windshield wipers is one of those common-sense impressions. Answer choice C is incorrect. Although the availability of a declarant as a witness can sometimes determine whether a hearsay statement is admissible, the fact that the declarant of the out-of-court statement is the person who is recounting it at trial does not prevent the statement from being inadmissible hearsay. Answer choice D is incorrect because the passenger's out-of-court statement, although clearly against the interest of the driver of the SUV, is not against the interest of the passenger. The statement was not against the passenger's proprietary or pecuniary interest, did not invalidate the passenger's claim against someone, and did not expose the passenger to civil or criminal liability. Moreover, in order for a declaration against interest to be admissible, the declarant must be unavailable. Here, the passenger is the person who made the out-of-court statement.

Question 4207 A plaintiff sued a forklift manufacturer in a product liability action after a forklift exploded when the propane tank powering the forklift detached from the body of the vehicle. In the action, the parties disputed the make and model of the forklift at issue. Without notice to the defendant forklift manufacturer, and without preliminary authenticating testimony, the plaintiff produced three sets of exhibits for admission into evidence at trial. The exhibits consisted of certified copies of a third-party retailer's business records identifying the make and model of the unit at issue (Exhibit A); a laser-etched metal plaque that had been affixed to the forklift that identified the make, model, and serial number of the unit (Exhibit B); and a certified copy of the license issued by a state agency identifying the forklift at issue (Exhibit C). The defendant objected to the admission of the exhibits into evidence on the ground that it should have been given written notice before the trial stating the plaintiff's intent to offer these items into evidence. How should the judge rule on the defendant's objection? Answers: Sustain the objection as to Exhibit A only. Sustain the objection as to Exhibit B only. Sustain the objection as to Exhibit C only. Sustain the objection as to all three exhibits.

Answer choice A is correct. The court will consider a number of items of evidence to be self-authenticating, meaning that they do not require extrinsic evidence of authenticity in order to be admitted. These include certified copies of public records, trade inscriptions (e.g., labels affixed in the course of business that indicate ownership), and business records. Generally, the proponent of a self-authenticating document is not required to give an adverse party advance notice of the intent to introduce the document. The proponent of a business record must, however, give an adverse party reasonable written notice prior to the trial or hearing of the intent to offer the record and must make the record available for inspection so that the party has a fair opportunity to challenge it. Here, the plaintiff should have sent the defendant notice of its intent to offer business records into evidence to give the defendant time to examine and prepare any appropriate challenges to the records. Answer choice B is incorrect because the metal plaque would be considered a self-authenticating trade inscription which does not require prior written notice before being offered into evidence at a hearing or trial. Answer choice C is incorrect because the signed, sealed public operating license likewise is a self-authenticating document that does not require prior written notice to the adverse party. Answer choice D is incorrect because only the business records in Exhibit A would require prior written notice to the adverse party.

Question 3027 A consumer filed a product liability action in federal court on the basis of diversity jurisdiction. In complying with a discovery request, the defendant's lawyer inadvertently—despite the lawyer's reasonable efforts to protect privileged material—included a report prepared at the lawyer's request by an investigator who was employed in anticipation of the litigation and was not expected to be called as a witness at trial. Immediately upon learning of the mistake, the defendant's lawyer sought to retrieve the report from opposing counsel, but the plaintiff's lawyer refused, asserting that the defendant's lawyer's inclusion of the report in the discovery material constituted a waiver of the protection for the material. The defendant's lawyer contended that the federal inadvertent waiver rule applied to this disclosure. Is the defendant's lawyer correct? Answers: Yes, because the federal inadvertent waiver rule applies to the disclosure of information covered by attorney work-product protection. Yes, because the federal inadvertent waiver rule applies to the disclosure of communications protected by the attorney-client privilege. No, because the federal inadvertent waiver rule applies only to disclosures made to a federal office or agency. No, because the federal inadvertent waiver rule does not apply to a federal case based on diversity jurisdiction.

Answer choice A is correct. The federal inadvertent waiver rule (Federal Rule 502) applies to the disclosure of information covered by the attorney work-product protection as well as communications covered by the attorney-client privilege. The report constitutes work product since it was prepared at the defendant's lawyer's request and in anticipation of the litigation. Answer choice B is incorrect because, although the inadvertent waiver rule does apply to disclosures of communications protected by the attorney-client privileges, the report was not protected by the attorney-client privilege because it was not a communication between the lawyer and his client. Answer choice C is incorrect because the inadvertent waiver rule applies to federal proceedings as well as to disclosures made to a federal office or agency. Answer choice D is incorrect because, although state rules regarding privileges are generally applied in a federal action based on diversity jurisdiction, the federal inadvertent waiver rule applies to a diversity case.

Question 1991 Several defendants, senior executives of a corporation, were charged with securities fraud. The government called as a witness another executive of the corporation, who had not been charged and who had been given immunity from prosecution, to authenticate handwritten notes that she had made after meetings of the corporation's management team at which the alleged fraud was discussed. The witness testified that she had prepared the notes on her own initiative to help her remember what had happened at the meetings. After this testimony, the government offered the notes into evidence to establish what had happened at the meetings. Should the witness's notes be admitted? Answers: No, because the notes are hearsay not within any exception. No, because the witness's immunity agreement with the government makes her notes untrustworthy and thus substantially more prejudicial than probative. Yes, because they are business records. Yes, because they are past recollections recorded.

Answer choice A is correct. The notes are hearsay because they are out-of-court statements offered to prove the truth of the matter asserted, and they do not fit any hearsay exception. Answer choice B is incorrect. The grant of immunity was given after the notes were made and therefore could not affect the reliability of the notes. Moreover, with the threat of prosecution removed, a witness is arguably more trustworthy. Answer choice C is incorrect. The notes do not qualify as business records because they were prepared on the witness's own initiative to help her remember what had happened at the meetings. The business records exception to the hearsay rule requires that it be a regular practice to make such records and that the records be kept in the course of regularly conducted activity. Answer choice D is incorrect because no foundation has been laid to admit the notes under the past recollection recorded exception to the hearsay rule. The executive has not testified that she has insufficient recollection to testify fully and accurately without the notes, nor has she testified that the notes are an accurate reflection of her knowledge when the matter was fresh in her memory. Moreover, even if the hearsay exception were satisfied, the notes themselves are not admissible. Under FRE 803(5), a record of recorded recollection "may be read into evidence but may not itself be received as an exhibit unless offered by an adverse party."

Question 4189 In a probate proceeding, the beneficiary of a purported holographic will sought to introduce a photocopy of that will as evidence of the contents of the missing original holographic will. An heir of the testator opposed the introduction of this photocopy into evidence. Contending that the testator was not of sound mind when the original will was written, the heir demanded that the beneficiary produce the original will. The applicable jurisdiction recognizes the validity of a holographic will and has adopted a dead man's statute. Is the photocopy of the will admissible without an explanation of the unavailability of the original? Answers: Yes, because the photocopy is a duplicate of the original. Yes, because the dead man's statute permits the introduction of the photocopy. No, because the will is a document that has legal effect. No, because there is question as to the validity of the will.

Answer choice A is correct. The photocopy of the will is a duplicate, since it was produced by a process that accurately produces the original. Consequently, it is admissible to the same extent as the original unless there is a genuine question as to the authenticity of the original or other circumstances that would make its introduction unfair. Since the only challenge advanced by the heir relates to the validity of the will for reasons that lie beyond the authenticity of the original, the photocopy is admissible. Answer choice B is incorrect because a dead man's statute, does not have a bearing on the admissibility of a photocopy of a testator's purported will in a probate proceeding. Answer choice C is incorrect because, although the will is a document that has legal effect and thus the original document rule applies, the introduction of a duplicate is permitted. Answer choice D is incorrect because, although the heir has raised a question as to the validity of the will, this question does not go to the authenticity of the original. Consequently, the photocopy of the will is admissible to the same extent as the original. Editor's Note: Remember that the Best Evidence Rule applies in narrow situations, which is what makes it an attractive and usually incorrect distractor. Before selecting the Best Evidence Rule as the correct answer, confirm that either the contents of the document are at issue or a witness is relying on the contents of the document when testifying. Your practice questions test concepts, including the Best Evidence Rule, in a variety of ways, in the event you see a question like it on the bar exam. You may see situations in which the Best Evidence Rule is the correct answer during your practice sessions, or on the bar exam.

Question 5912 A plaintiff brought an action for the tort of battery against a defendant. At trial, the defendant, asserting self-defense, introduced the deposition testimony of a witness, in which the witness stated that the plaintiff had initiated the altercation with the defendant. The witness was unavailable to testify at the trial because he began suffering from a major mental disability after his earlier deposition. The plaintiff then called a friend of the witness who testified that the witness was employed by the defendant. The friend also testified that the witness had a reputation in the community for being untruthful. The defendant objected to the friend's testimony. Which action should the court take? Answers: Admit the friend's entire testimony regarding the witness. Admit the testimony regarding the witness's employment only. Admit the testimony regarding the witness's character for untruthfulness only. Exclude the friend's entire testimony regarding the witness.

Answer choice A is correct. When a hearsay statement is admitted into evidence, the credibility of the declarant may be attacked (and, if attacked, supported) by any evidence that would be admissible if the declarant had testified as a witness. The declarant need not be given the opportunity to explain or deny any inconsistent statement or conduct, whether such statement or conduct occurred before or after the hearsay statement. In this case, the plaintiff could properly call the friend to impeach the witness's prior deposition testimony, even though the witness was not available to testify at trial. The friend's testimony regarding the fact that the witness was employed by the defendant is relevant to impeach the witness by evidence of possible bias, and thus admissible. Bias is always relevant to the credibility of a witness's testimony. The friend's testimony about the witness's reputation in the community for being untruthful is also admissible because a witness's credibility may be attacked by testimony (reputation or opinion) regarding the witness's character for untruthfulness. Therefore, the friend's entire testimony should be admitted to impeach the witness. Answer choice B is incorrect because the witness may also be impeached by testimony regarding his character for untruthfulness. Answer choice C is incorrect because the friend's testimony about the witness's employment, relevant to bias, can be admitted to impeach. Answer choice D is incorrect because both parts of the friend's testimony constitute proper impeachment evidence.

Question 6494 A defendant has been sued in a civil action for battery. The plaintiff plans to introduce a letter from the defendant to his attorney in which the defendant summarized his account of the incident. The letter had been inadvertently disclosed to the plaintiff in response to a discovery request, despite the defense attorney's reasonable precautions against such disclosure and prompt efforts to rectify the error. Before sending the letter, the defendant had asked his sister to proofread it for him. The defendant plans to invoke his Fifth Amendment Right Against Self-Incrimination if called by the plaintiff to testify at trial. On grounds of the attorney-client privilege, the defense attorney has made a motion in limine to prevent the plaintiff from introducing this letter at trial. The plaintiff contends that he has a substantial need for the letter, and cannot obtain the information within the letter by any other means, as the defendant's sister is now deceased. Of the following, which would provide the strongest support for denying the defense attorney's motion? Answers: The defendant will not testify due to his invocation of his Fifth Amendment right not to testify. The defendant showed the letter to his sister before sending it to his attorney. The letter was inadvertently disclosed to the plaintiff. The plaintiff has a substantial need for the letter, and cannot obtain the information in the letter by any other means.

Answer choice B is correct. A confidential communication between a client and an attorney for the purpose of seeking legal advice or representation is privileged. However, the communication must be intended to be confidential in order to be privileged. If the defendant showed the letter to his sister before sending it to the attorney, the confidentiality of the letter is called into question, and the privilege may not apply. Answer choice A is incorrect because the defendant's unavailability at trial does not give the plaintiff the right to use a letter protected by the attorney-client privilege. Answer choice C is incorrect because the inadvertent disclosure of a privileged communication or information does not necessarily waive the attorney-client privilege if the holder of the privilege took reasonable steps to prevent disclosure and promptly took reasonable steps to rectify the error. Answer choice D is incorrect because this is the standard for compelling the production of attorney work product, not a confidential communication protected by the attorney-client privilege.

Question 4319 A defendant was charged with battery following a bar fight with his neighbor. At trial, the defendant asserted that he did not initiate the altercation, but instead acted in self-defense. In addition to testifying about the event in question, he also sought to testify that the preceding night, he and a co-worker had gone out for a drink at the same bar, and that the evening passed peacefully. Prior to his own testimony, the defendant sought to introduce testimony of a lifelong acquaintance of the neighbor that, in the opinion of the acquaintance, the neighbor had a violent streak. After his testimony, the defendant sought to introduce testimony of the pastor of the church that the defendant regularly attended that the defendant had a reputation among the members of the church as a nonviolent person. Following testimony introduced by the prosecution that impeached the defendant's truthfulness, the defendant sought to introduce testimony of his employer that, in his opinion, the defendant was a truthful individual. Which of the proffered testimony is most likely to be successfully challenged by the prosecution? Answers: The testimony of the lifelong acquaintance of the neighbor regarding the neighbor's violent streak. The testimony of the defendant regarding his peaceful behavior on the night before the bar fight. The testimony of the defendant's pastor as to the defendant's reputation as a nonviolent person. The testimony of the defendant's employer that the defendant was a truthful individual.

Answer choice B is correct. A criminal defendant may not introduce specific instances of his behavior to prove a relevant character trait when that character trait is being used to prove that the defendant acted in conformity with it on the night in question. Although a defendant's character for peacefulness is relevant with regard to the defendant's contention that he acted in self-defense, this character trait cannot be established by specific events. Answer choice A is incorrect because a defendant may introduce evidence of the victim's character when it is relevant to the defense asserted though opinion testimony. The neighbor's character for violence is relevant to the defendant's position that the defendant did not initiate the fight. Answer choice C is incorrect because a defendant may introduce reputation as well as opinion testimony as to his nonviolent character in order to advance his self-defense position. The reputation of the defendant may be based on a group of people with whom the defendant regularly engages, such as members of his church. Answer choice D is incorrect because a defendant, after testifying, may introduce evidence of his character for truthfulness if it has been impeached by the prosecution. Evidence in the form of opinion as well as reputation testimony is permitted.

Question 4319 A defendant was charged with battery following a bar fight with his neighbor. At trial, the defendant asserted that he did not initiate the altercation, but instead acted in self-defense. In addition to testifying about the event in question, he also sought to testify that the preceding night, he and a co-worker had gone out for a drink at the same bar, and that the evening passed peacefully. Prior to his own testimony, the defendant sought to introduce testimony of a lifelong acquaintance of the neighbor that, in the opinion of the acquaintance, the neighbor had a violent streak. After his testimony, the defendant sought to introduce testimony of the pastor of the church that the defendant regularly attended that the defendant had a reputation among the members of the church as a nonviolent person. Following testimony introduced by the prosecution that impeached the defendant's truthfulness, the defendant sought to introduce testimony of his employer that, in his opinion, the defendant was a truthful individual. Which of the proffered testimony is most likely to be successfully challenged by the prosecution? Answers: The testimony of the lifelong acquaintance of the neighbor regarding the neighbor's violent streak. The testimony of the defendant regarding his peaceful behavior on the night before the bar fight. The testimony of the defendant's pastor as to the defendant's reputation as a nonviolent person. The testimony of the defendant's employer that the defendant was a truthful individual.

Answer choice B is correct. A criminal defendant may not introduce specific instances of his behavior to prove a relevant character trait when that character trait is being used to prove that the defendant acted in conformity with it on the night in question. Although a defendant's character for peacefulness is relevant with regard to the defendant's contention that he acted in self-defense, this character trait cannot be established by specific events. Answer choice A is incorrect because a defendant may introduce evidence of the victim's character when it is relevant to the defense asserted though opinion testimony. The neighbor's character for violence is relevant to the defendant's position that the defendant did not initiate the fight. Answer choice C is incorrect because a defendant may introduce reputation as well as opinion testimony as to his nonviolent character in order to advance his self-defense position. The reputation of the defendant may be based on a group of people with whom the defendant regularly engages, such as members of his church. Answer choice D is incorrect because a defendant, after testifying, may introduce evidence of his character for truthfulness if it has been impeached by the prosecution. Evidence in the form of opinion as well as reputation testimony is permitted.

Question 6261 A son and a daughter are opposing parties in federal court. At trial, the daughter presented evidence that her father has been missing for ten years, and that no one has heard from him in that time. The son testified that he received a phone call three years ago from a person that he believes was his father. In the jurisdiction, a rebuttable presumption arises that a person is dead when a party establishes that the person has been missing and not heard from for more than seven years. Which of the following is correct? Answers: The jury must find that the father is dead. The jury may find that the father is dead. The burden has shifted to the son to persuade the jury that the father is alive. The judge must instruct the jury to conclude that the father is dead.

Answer choice B is correct. A presumption is a conclusion that the trier of fact is required to draw upon a party's proof of an underlying fact or set of facts (i.e., basic facts). A rebuttable presumption shifts the burden of production, but not the burden of persuasion, to the opposing party. However, a rebuttable presumption may be overcome by evidence to the contrary. If no contrary evidence is introduced, the judge must instruct the jury to accept the presumption. If contrary evidence is introduced, as is the case here, then the presumption no longer has a preclusive effect. At this point, the jury may, but is not required to, draw the conclusion from the basic facts. Thus, the jury may determine the weight and credibility of all of the evidence. For this reason, answer choice A is incorrect. Answer choice C is incorrect because a rebuttable presumption shifts the burden of production, but not the burden of persuasion, to the opposing party. Answer choice D is incorrect because it is an incorrect statement of law. After a rebuttable presumption no longer has a preclusive effect due to the introduction of contrary evidence, a judge may instruct the jury that it may, but is not required to, draw the conclusion from the basic facts.

Question 6508 A defendant is on trial for embezzling $50,000 from his former employer. The prosecution wishes to offer into evidence an anonymous letter, in its entirety, that was received by the defendant's former supervisor. The former supervisor testified that the letter was written in the defendant's handwriting, which the supervisor knew from their years of working together. The letter reads "I am consumed by guilt for what I have done. Here is half the money I took from you, and if you promise not to prosecute, I will send you the rest later this year. If you accept this arrangement, please post a personal ad in the local paper using the phrase 'All is forgiven,'" and was accompanied by $25,000 in cash. The defense objects to the admission of the letter. Is the letter admissible? Answers: Yes, under the hearsay exception for statements of the declarant's present intent, motive, or plan. Yes, because as a statement by a party to the current litigation, it is not hearsay. No, because the letter has not been properly authenticated. No, because public policy calls for the exclusion of statements made in a negotiation to settle a claim.

Answer choice B is correct. A statement made by a party to the current litigation is not hearsay if it is offered by an opposing party. Because the letter has been authenticated by the defendant's previous supervisor, the letter is admissible. Answer choice A is incorrect. Although a statement of the declarant's then-existing state of mind qualifies as an exception to the hearsay rule, this exception only applies to a statement of present intent, motive, or plan that is used to prove conduct in conformity with that state of mind. In the letter, although the author discusses future actions as well, the author's admission of having taken money from his former employer relates to a past event, not a future one. Consequently, the admission in this letter is not admissible under this hearsay exception. Answer choice C is incorrect because a lay witness with personal knowledge of the claimed author's handwriting may testify as to whether the document is in that person's handwriting, thus authenticating it. Answer choice D is incorrect. Compromise offers made by any party, as well as any conduct or statements made during compromise negotiations, are not admissible to prove or disprove the validity or amount of a disputed claim. Here, the defendant is not disputing either the validity or the amount of the claim. He is admitting that he embezzled the full amount (i.e., $50,000) and is trying to avoid prosecution by negotiating over the return of the remaining half (i.e., $25,000).

Question 6272 A defendant, his cousin, and a friend planned to rob a convenience store. The friend watched the defendant and his cousin put on ski masks and enter the convenience store. Moments later, the friend heard a gunshot, and the defendant fled. The cousin stumbled out of the convenience store, pointed at his bleeding foot, and told the accomplice, "He shot me! The moron dropped his gun and shot me, I'm going to kill that idiot!" The cousin ran after the defendant, and the friend called the police. The police found the cousin dead from a bullet wound to the chest and charged the defendant with murder. At trial, the defendant wants to introduce the cousin's statement to the friend as evidence that the cousin was the initial aggressor. The prosecution objects. What is the defendant's best argument for finding that the statement is admissible? Answers: The declarant is unavailable as a witness. The statement indicates the cousin's present intent. The statement was a dying declaration. The statement was made by a co-conspirator.

Answer choice B is correct. A statement of present intent, motive, or plan can be used to prove conduct in conformity with that state of mind. Here, the defendant is trying to offer the statement to show that his cousin intended to attack him to help establish his defense. Because a statement of the declarant's then-existing state of mind falls within a hearsay exception, this is the defendant's best argument. Answer choice A is incorrect because a declarant is not deemed unavailable if the unavailability is due to the procurement or wrongdoing of the proponent of the statement in order to prevent the declarant from testifying at or attending the trial. Because of the possibility that the defendant killed his cousin in order to silence him at trial, this is not the defendant's strongest argument. Answer choice C is incorrect because it would be very difficult to argue that the cousin believed his death was imminent from a gunshot wound to his foot at the time he made the statement. This is evidenced by the fact that he ran after the defendant and made no statement indicating that he believed his death was imminent. Answer choice D is incorrect because, although a statement made by a co-conspirator during and in furtherance of a conspiracy is admissible as an opposing party's statement against other co-conspirators, the cousin's outburst was not made in furtherance of the already failed robbery.

Question 7260 A plaintiff sued a defendant for libel after the defendant published an article on his website calling the plaintiff an adulterer. At trial, the defendant's attorney called the plaintiff's wife who testified that on two separate occasions, the wife had found the plaintiff in a hotel with a prostitute. The plaintiff objected to the wife's testimony. Should the court sustain the plaintiff's objection? Answers: No, because character may be proven by specific instances of conduct in civil cases. No, because the plaintiff's infidelity is directly at issue in the trial. Yes, because the wife's testimony is improper character evidence. Yes, because the wife's testimony is barred by spousal privilege.

Answer choice B is correct. In a civil case, evidence of a person's character (or character trait) generally is inadmissible to prove that the person acted in accordance with that character (or character trait) on a particular occasion. Character evidence is admissible, however, when character is an essential element of a claim or defense, rather than a means of proving a person's conduct. Character is most commonly an essential element in defamation cases (character of the plaintiff). When character evidence is admissible as evidence in a civil case, it may be proved by specific instances of a person's conduct as well as either by testimony about the person's reputation or by testimony in the form of an opinion. Here, because this action is for defamation, the plaintiff's character is an essential element of the claim and therefore, specific instances of the plaintiff's infidelity are admissible. Answer choice A is incorrect because it overstates the law. In civil cases, specific instances of conduct may be used to prove character only when character is an essential element of a claim or defense. Answer choice C is incorrect because specific instances of the plaintiff's infidelity are proper character evidence in this case. Answer choice D is incorrect. "Spousal privilege" comprises two distinct privileges: spousal immunity and confidential marital communications. Neither applies here; spousal immunity does not apply in civil cases, and the wife's testimony does not involve a confidential communication made in reliance on the sanctity of marriage.

Question 5914 A pedestrian was struck by a school bus while crossing a major intersection. The pedestrian brought suit against the bus company for negligence. At trial, the pedestrian testified that he always waits for the pedestrian crossing signal before crossing major intersections. On cross-examination, the bus company asked the pedestrian about a ticket he had received for jaywalking at a major intersection three months before the accident in question. Is the bus company's question likely permissible? Answers: Yes, because the prior ticket demonstrates that the pedestrian did not wait for the crossing signal when the accident occurred. Yes, because the evidence contradicts the pedestrian's testimony that he always waits for the pedestrian crossing signal. No, because the receipt of a ticket for jaywalking is not probative of the truthfulness of the recipient of the ticket. No, because the probative value of the evidence is substantially outweighed by the danger of unfair prejudice.

Answer choice B is correct. A witness may be impeached by evidence that contradicts the witness's testimony. In this case, the pedestrian testified that he always waits for the pedestrian crossing signal before crossing major intersections. The bus company's question about the prior ticket for jaywalking directly contradicts the pedestrian's testimony. Therefore, the question should be permitted. Answer choice A is incorrect. The prior ticket likely cannot be used to prove that the pedestrian must not have waited for the crossing signal in this instance because he illegally crossed a major intersection in the past. Evidence of a specific act is not admissible to prove a person's character in order to show that the person acted in accordance with that character on a particular occasion. Answer choice C is incorrect. While the receipt of a ticket for jaywalking does not in and of itself relate to the truthfulness of the recipient, in this instance the ticket calls into question the plaintiff's truthfulness with regard to his statement that he always waits for the pedestrian crossing signal. Answer choice D is incorrect because the probative value of the ticket, which directly conflicts with the pedestrian's testimony, is likely not substantially outweighed by the danger of unfair prejudice. "Substantially outweighed" is a high standard to meet.

Question 6971 A girlfriend testified on behalf of her boyfriend when he was prosecuted for first-degree murder of his ex-wife. When questioned by the defense, the girlfriend testified that the ex-wife had provoked her boyfriend by telling him she had cheated on him throughout their marriage. In a momentary heat of passion, the boyfriend strangled the ex-wife. On cross-examination, the prosecution asked if the girlfriend had ever underreported her annual income on her tax forms. When the girlfriend denied doing so, the prosecution sought to introduce evidence of the girlfriend's annual income and tax forms from the previous three years, all of which showed that the girlfriend underreported her earnings for tax purposes. Is this evidence admissible? Answers: No, because a specific instance of conduct is not admissible to attack or support the girlfriend's character for truthfulness. No, because the fact that the girlfriend may have underreported her annual income on her tax forms cannot be proven through extrinsic evidence. Yes, because the fact that the girlfriend may have underreported her annual income on her tax forms is probative of her untruthfulness. Yes, because the probative value of the evidence is not substantially outweighed by the unfair prejudice that it may cause.

Answer choice B is correct. Although specific instances of conduct are generally not admissible to attack or support the witness's character for truthfulness, on cross-examination, a witness may be asked about specific instances of conduct if it is probative of the truthfulness or untruthfulness of the witness (or another witness about whose character the witness being cross-examined has testified). When the witness denies a specific instance of conduct on cross-examination, extrinsic evidence is not admissible to prove that instance in order to attack or support the witness's character for truthfulness. In this case, the prosecution was permitted to question the girlfriend on cross-examination about underreporting her annual income on her tax forms, but the extrinsic evidence it attempted to submit to prove the underreporting is not admissible. Answer choice A, although a true statement of law, is not applicable when the witness is questioned on cross-examination about a specific instance of conduct. Answer choice C is incorrect because the girlfriend's underreporting of her income, although probative of her truthfulness, cannot be proven by extrinsic evidence. Answer choice D is incorrect. The central issue here is that the prosecution is attempting to impeach the girlfriend by using a specific instance of conduct to attack her character for truthfulness. The fact that she lied on her tax forms has probative value that is not outweighed by any unfair prejudice. However, extrinsic evidence is not admissible to prove a specific instance of conduct on cross-examination for the purpose of impeaching a witness's character for truthfulness, so the evidence is not admissible.

Question 6263 At his trial for larceny, a defendant called his brother as a character witness. The brother testified that the defendant had a reputation in the community for being an honest man. During rebuttal, the prosecutor called the defendant's former employer to testify that the defendant lied on his job application. The defense attorney objected, arguing that the testimony is an improper use of character evidence. How should the judge rule on the defense attorney's objection? Answers: Sustain the objection, because the evidence has no probative value on any issue in the case. Sustain the objection, because the evidence can be inquired into only during cross-examination of the brother. Overrule the objection, because the defense has "opened the door" to the prosecutor's admission of this evidence. Overrule the objection, because the evidence offered involves dishonesty.

Answer choice B is correct. Although the prosecution generally cannot introduce evidence of a defendant's bad character, the defendant can make his character an issue in the case by offering evidence of his good character. When the defendant "opens the door" in this way, the prosecution is free to rebut the defendant's claims by attacking the defendant's character. In introducing evidence as to the defendant's character, the prosecution is generally limited to the same type of evidence that the defendant offers. However, on cross-examination, the prosecution may question a defendant's character witness about specific instances of the defendant's conduct. Here, the prosecution could only introduce the specific instance of conduct by asking the brother about the conduct on cross-examination. This evidence cannot be introduced by extrinsic evidence, i.e., the testimony of the defendant's former employer. Answer choice A is incorrect because this character evidence would certainly be relevant, but it should be excluded because it was not introduced properly. Answer choice C is incorrect. Although the defendant "opened the door" by having his brother testify as to his character, thus permitting the prosecution to rebut that testimony, the prosecution is limited to asking the brother about the specific instance of conduct on cross-examination, and cannot introduce extrinsic evidence of the defendant lying on a job application. Answer choice D is incorrect. The fact that the evidence offered involves dishonesty allows the prosecution to introduce the evidence to rebut the brother's testimony that the defendant has a reputation for being an honest man. However, the prosecution can only introduce this evidence by questioning the brother on cross-examination.

Question 643 The defendant's attorney in a fraud case called a witness to testify as to the defendant's character. On cross-examination, the prosecutor asked the witness whether he had ever been arrested for writing bad checks. In fact, the witness had been arrested two years ago for writing bad checks, but the charges had been dropped due to a lack of evidence that the witness had committed the crime. The defendant's attorney objects to the question. Should the prosecutor be allowed to ask the question? Answers: No, because a witness who is not the defendant may not be cross-examined about prior bad acts. No, because the witness was never convicted of the crime. Yes, because writing bad checks is probative of the witness's untruthfulness. Yes, because the arrest occurred only two years ago.

Answer choice B is correct. An arrest for a bad act is not a bad act itself. Therefore, a witness may not be cross-examined about an arrest. In this case, there was no evidence that the witness actually wrote any bad checks; as such, the subject may not be brought up on cross-examination. Answer choice A is incorrect because a witness may sometimes be asked on cross-examination about specific instances of conduct, including a prior bad act. There are some limitations, however: the judge must determine that the specific instance of conduct is probative of untruthfulness, and the attorney must have a good faith basis for asking about the conduct. Answer choice C is incorrect because, even though actually writing bad checks would be probative of the witness's truthfulness, simply being arrested is not probative of truthfulness. Answer choice D is incorrect because (1) time limits are relevant when a witness is being impeached with evidence that he has been convicted of a crime, not with evidence of specific instances of conduct; and (2) as previously mentioned, being arrested is not probative of truthfulness.

Question 6509 A defendant charged with aggravated assault of a hiker who wandered through his campsite in a national park has alleged that he shot at the hiker in self-defense. The defendant testified that when the hiker crossed the campsite at dusk, the defendant thought that the hiker intended to rob him. The prosecution wishes to call a bartender to testify that the year before, the defendant had fired at another patron of the bar, then publicly claimed that he had done so in self-defense, while privately admitting to the bartender that he done so simply for the fun of it. Is the bartender's testimony admissible? Answers: No, because the conduct does not relate to an essential element of the crime charged. No, because the testimony presents improper character evidence. Yes, as proper impeachment evidence. Yes, because it constitutes habit evidence.

Answer choice B is correct. Evidence of a specific act is not admissible to prove a person's character in order to show that the person acted in accordance with that character trait on a particular occasion. Here, the prosecution is attempting to show that the defendant, having lied in the past about acting in self-defense, is lying now. Answer choice A is incorrect because the bartender's testimony need not establish an element of aggravated assault with respect to the event at issue, as the testimony is relevant to the defendant's assertion that he acted in self-defense in shooting at the hiker. However, the testimony constitutes inadmissible character evidence. Answer choice C is incorrect because a specific instance of a witness's past conduct is not admissible as extrinsic evidence to attack or support the witness's credibility. Such conduct can only be asked about on cross-examination of the witness. Because this testimony does not contradict the defendant's testimony, it is inadmissible in this context as impeachment evidence. Answer choice D is incorrect. Although habit evidence is admissible, this testimony about a single prior alleged action taken by the defendant does not constitute a particular routine reaction to a specific set of circumstances.

Question 1962 A defendant was charged with robbery of a savings and loan branch after being arrested near the scene and found with marked bills. An hour after the robbery, the officer investigating the crime videotaped an interview with an eyewitness, in which the eyewitness described the crime and the robber. The officer then arranged for a lineup, at which the teller who was robbed identified the defendant as the robber. The officer later obtained computerized records of that day's deposits and withdrawals at the savings and loan, which allowed the calculation of how much cash was taken in the robbery. A month later, the teller testified before a grand jury, which indicted the defendant. The teller and the eyewitness both died of unrelated causes shortly afterward. At trial, which of the following evidence, if properly authenticated, may properly be admitted against the defendant over his attorney's objection that its receipt would violate the confrontation clause? Answers: A transcript of the teller's sworn grand jury testimony. The computerized records from the savings and loan. The officer's testimony that the teller picked the defendant out of the lineup as the robber. The videotape of the eyewitness's statement.

Answer choice B is correct. The computerized records are not a testimonial statement because they were not prepared for the purpose of being used in a criminal prosecution. The Crawford case specifically cites business records as an example of statements that are generally not testimonial. Answer choice A is incorrect. The teller's grand jury testimony is a classic testimonial statement barred by the confrontation clause. Answer choice C is incorrect. The officer's statement does not violate the confrontation clause because he can be cross-examined, but the teller's identification of the defendant is a testimonial statement because it was made for the purpose of being used in a criminal prosecution. Since the teller is dead, it would violate the defendant's right of confrontation to admit the statement of identification by the teller. Answer choice D is incorrect. The eyewitness's statement, made to a police officer, is clearly testimonial. Because the eyewitness was not cross-examined when the statement was made and cannot be cross-examined at trial (as the eyewitness is dead), the statement is inadmissible under the confrontation clause.

Question 6507 A husband is on trial in a federal court for various charges related to organized crime. The federal prosecutor has arranged for the husband's wife to testify against her husband in exchange for leniency in an unrelated case against her. At trial the prosecutor calls the wife, who has been granted immunity from prosecution, to testify that she heard her husband discussing a planned drug deal on the phone a week before he was arrested. Can the prosecutor call the wife to testify in this matter against her husband? Answers: The wife can be required to be a witness and to testify against her husband. While the prosecutor can call the wife to testify, the wife can refuse to appear as a witness against her husband. Although the wife can be called as a witness, she cannot testify against the husband over his objection. No, the wife cannot be called as a witness against her husband.

Answer choice B is correct. The general rule is that the spouse of a criminal defendant may not be called as a witness by the prosecution. Nor may a married person be compelled to testify against his spouse in any criminal proceeding. In federal courts (and a majority of states), the witness-spouse holds the privilege and may choose to testify but cannot be compelled to do so. Answer choice A is incorrect because the wife cannot be required to testify against her husband in federal court. Answer choice C is incorrect because the wife, not the husband, is the holder of this privilege in federal court. Answer choice D is incorrect because the wife can be called, if she consents to testify.

Question 6499 A defendant is on trial for armed robbery and felony murder. The prosecution seeks to admit testimony by the first witness to arrive at the scene of the crime. The witness discovered the victim just before he died of a gunshot wound, and the victim identified the defendant as his assailant only moments before he died. The defense asserts that the victim was too delirious from blood loss to know that he was dying, and hopes to present a statement from the victim's widow to support this assertion. However, both parties agree that the widow's statement is privileged under federal law. In what manner should the court determine whether the victim's statement is a dying declaration? Answers: Allow the prosecution to admit the testimony only if the judge determines that the witness is credible. Consider only the unprivileged evidence from both sides outside the presence of the jury. Consider all of the evidence from both sides outside the presence of the jury. Allow only the unprivileged evidence from both sides at the jury trial so the jury may decide whether the victim believed he was dying.

Answer choice B is correct. The trial judge generally decides preliminary questions regarding the competency of evidence, including the admissibility of evidence, whether privilege exists, and whether a person is qualified to be a witness. Hearings on preliminary matters must be conducted outside the presence of the jury when the hearing involves the admissibility of confessions, when a defendant in a criminal case is a witness and so requests, or when justice requires it. Here, the determination of the admissibility of a statement by the victim identifying the defendant as his assailant is clearly a matter that justice requires that the jury not be present. The court is not bound by the Federal Rules in deciding these questions, except with respect to privileges. Accordingly, even the judge cannot consider the privileged evidence. Answer choice A is incorrect because determinations of credibility must be made by a jury. Answer choice C is incorrect because the court is bound by the Federal Rules in deciding preliminary questions of admissibility with respect to privileges. Answer choice D is incorrect because this preliminary question must be decided by the judge, not the jury.

Question 3031 An artist entered into a written agreement to sell a patron a partially finished painting once it was complete. The patron later learned that the artist planned to sell the painting to a third party who offered to pay the artist more than the contract price. The patron filed suit to compel the artist to sell the painting to her in accord with the terms of their agreement, while the artist denied that the painting the artist planned to sell to the third party was the subject of the agreement with the patron. At trial, the patron did not introduce the written agreement or explain its absence. Rather, the patron sought to testify that, when she signed the agreement, the artist had pointed to the painting in question and stated that it was the patron's painting. The artist's attorney objected to the testimony that the artist identified the painting as belonging to the patron. How should the court rule? Answers: Sustain the objection, because the artist's statement was hearsay. Sustain the objection, because the patron failed to produce the written agreement or explain its absence. Overrule the objection, because the statement is not hearsay. Overrule the objection, because the statement was relevant.

Answer choice B is correct. Under the best evidence rule, the original document or a reliable duplicate must be used to prove the contents of a writing unless its absence is satisfactorily explained. Here, the plaintiff intends to introduce the statement to prove that the written agreement refers to a specific painting without introducing the written agreement itself. Consequently, the patron may not introduce that statement to establish that the painting in question was the painting that was the subject of the agreement. Answer choice A is incorrect because, although the artist's statement was being introduced for its truth, it is non-hearsay as a statement of a party to the action. Answer choice C is incorrect because, although the artist's oral statement was non-hearsay, the best evidence rule prevents its admission into evidence. Answer choice D is incorrect because, while the statement meets the test for relevancy, in that it is both probative and material, it is inadmissible on other grounds. Editor's Note: Remember that the Best Evidence Rule applies in narrow situations, which is what makes it an attractive and usually incorrect distractor. Before selecting the Best Evidence Rule as the correct answer, confirm that either the contents of the document are at issue or a witness is relying on the contents of the document when testifying. Your practice questions test concepts, including the Best Evidence Rule, in a variety of ways, in the event you see a question like it on the bar exam. You may see situations in which the Best Evidence Rule is the correct answer during your practice sessions, or on the bar exam.

Question 5841 A defendant was on trial for the attempted murder of his employer. The prosecutor called the defendant's wife to the witness stand and asked her to recall anything the defendant said prior to leaving their home on the morning that the attempted murder took place. The wife testified that while they were in bed, the defendant stated, "I've had enough of the way my employer treats me. Something has to be done." The defendant objected to his wife's testimony. The defendant and his wife had legally separated after the defendant was arrested. Is the wife's testimony admissible? Answers: No, because the defendant can prohibit his wife from testifying against him at all. No, because the wife's testimony concerns a confidential communication. Yes, because the defendant and his wife were legally separated. Yes, because the wife may choose to testify against the defendant.

Answer choice B is correct. Under the confidential marital communications privilege, a communication made between spouses while they were married is privileged if the communication was made in reliance on the sanctity of marriage. The privilege is held by both spouses and applies in both civil and criminal cases. Under the majority view, either spouse may assert the privilege and refuse to testify about the communication or prevent the other spouse from testifying. The time for asserting this privilege extends beyond the termination of the marriage. In this case, the wife testified about a statement the defendant made while the wife and defendant were in bed, during their marriage. Thus, the defendant's statement likely falls under the confidential marital communications privilege and is therefore inadmissible. Answer choice A is incorrect. In a criminal case, the witness-spouse holds the spousal immunity privilege and may choose to testify, but cannot be compelled to do so. Here, the wife can choose to testify; the defendant cannot prevent her from taking the stand against him. However, the wife is prohibited from testifying as to confidential marital communications. Answer choice C is incorrect. The confidential marital communications privilege continues to apply even after the termination of the marriage to communications made during the marriage. Furthermore, a legal separation does not terminate a marriage; there must be an annulment or divorce. Answer choice D is incorrect. Although the wife may choose to testify against the defendant, she may not testify as to any confidential marital communications.

Question 6506 A church is being sued for negligent hiring after their bus driver got in a drunk driving accident while driving a bus full of parishioners to a church retreat. Evidence discovered by the plaintiffs suggests that the bus driver had a well-documented drinking problem. The defense called a witness from the bus driver's last employer, who testified that no one at the driver's last job was aware that the bus driver had a drinking problem. The plaintiff's attorney asked on cross-examination whether the witness is a member of the church. The defense objects to the question as an improper impeachment question. Is the plaintiff's attorney's question proper? Answers: No, because evidence of a witness's religious belief is not admissible to impeach the witness's credibility. No, because it would violate the witness's First Amendment rights. Yes, as proper impeachment evidence. Yes, because the witness's religious belief is relevant to her credibility under oath.

Answer choice C is correct. Because a witness may be influenced by his relationship to a party (e.g., employment), his interest in testifying (e.g., avoidance of prosecution), or his interest in the outcome of the case (e.g., receipt of an inheritance), a witness's bias or interest is always relevant to the credibility of his testimony, and a witness may always be impeached on this ground. Because the witness's affiliation with the church may influence her to give testimony that is favorable to the church, this is a proper impeachment inquiry. Answer choice A is incorrect. Although her religious belief might not be relevant, whether the witness is a member of the defendant church is relevant to her potential bias. Answer choice B is incorrect because the First Amendment does not prohibit the questioning of the witness regarding her church membership in order to impeach her testimony when the church is a party to the case. Answer choice D is incorrect because evidence of a witness's religious opinions or beliefs is not admissible to attack or support a witness's credibility. However, such evidence may be admissible to show bias or interest, such as when the witness is affiliated with a church that is a party to a lawsuit.

Question 7532 While driving through her neighborhood, a woman was involved in a car accident with her neighbor. The neighbor alleged that the woman failed to stop at a stop sign. The neighbor brought an action for negligence against the woman. At trial, the woman intends to call a witness to the stand to testify that the woman stops at the stop sign all the time. The witness is the woman's coworker and frequently carpools with the woman to work, driving on a route with the same stop sign. However, the witness was not present for the accident with the neighbor. Should the court admit the witness's testimony? Answers: No, because the witness did not observe the accident. No, because the testimony is improper character evidence. Yes, because it can be used to prove the woman stopped at the stop sign on the day of the accident. Yes, because the woman's character for careful driving is an essential element of her defense.

Answer choice C is correct. Evidence of a person's habit or an organization's routine is admissible to prove that the person or organization acted in accordance with the habit or routine on a particular occasion. A habit is a person's particular routine reaction to a specific set of circumstances, and may be admitted without corroboration and without an eyewitness. Habit evidence must be specific. In this case, the woman intends to call a witness to testify that the woman has stops at this particular stop sign all the time. The witness's testimony that the woman stops at this stop sign on their commutes to work is likely to be considered habit evidence, and can be offered to prove the woman stopped at the stop sign on the day of the accident. Answer choice A is incorrect because a witness testifying as to a person's habit need not have observed the incident or event at issue. Habit evidence may be admitted without corroboration and without an eyewitness. Answer choice B is incorrect because the witness's testimony is specific enough to constitute habit evidence, and is not merely character evidence. Accordingly, it can be used to prove that the woman stopped at the stop sign on the day of the accident. Answer choice D is incorrect because the woman's character is not an essential element in her defense that she stopped at the stop sign. The woman's character is not at issue in this negligence action.

Question 6976 A man was prosecuted for the false imprisonment of a woman after he allegedly pretended to have a broken arm, asked the woman to help him carry a box into the back of his van, and then pushed her into the van and locked it. At trial, the prosecution attempted to introduce evidence during a female witness's direct examination that five years ago, the man had impersonated a policeman, entered the witness's home, and made serious threats of harm if the witness did not stay in the home and answer his questions. Is this evidence admissible? Answers: Yes, because it shows that the man has the propensity to falsely imprison women. Yes, because it is relevant evidence that shows the man's preparation and planning. No, because it is improper character evidence. No, because it can only be introduced during cross-examination.

Answer choice C is correct. Evidence of a specific act is not admissible to prove a person's character in order to show that the person acted in accordance with that character on a particular occasion. Here, the man's conduct is being admitted to prove that the man acted in accordance with his criminal propensity to falsely imprison women. Therefore, the evidence is inadmissible. Answer choice A is incorrect because character evidence cannot be admitted to show a defendant's criminal propensity in order to prove that he committed the crime for which he is charged. Answer choice B is incorrect. Although a defendant's crimes or other wrongful acts are not admissible to show his criminal propensity in order to prove that he committed the crime for which he is charged, such bad acts are admissible for another purpose, such as proving motive, opportunity, intent, preparation, plan, knowledge, identity, absence of mistake, or lack of accident. Here, the circumstances of the man's prior bad act are so unrelated to the current charge of false imprisonment that they do not tend to prove preparation or common plan. The only common denominator in both bad acts is that the man falsely imprisoned women. Answer choice D is incorrect. When a character witness is cross-examined, the court may allow a party to inquire into specific acts committed by the person about whom the witness is testifying. However, even on cross-examination, specific acts cannot be admitted to prove that, because the defendant had a propensity to commit crimes, the defendant committed the charged crime. Therefore, this evidence would also have been inadmissible during cross-examination.

Question 3022 In a sexual harassment action brought by an employee against her employer, the employee alleged that her supervisor had created a hostile work environment by making repeated crude and explicit sexual comments that were unwelcome. The employer filed a motion to admit evidence that the employee had a sexual relationship with her previous supervisor to show that the employee welcomed the employer's advances. How should the court rule on the admissibility of this evidence? Answers: The court should refuse to admit evidence of the relationship because evidence offered to prove a victim's sexual conduct or predisposition is inadmissible in a civil case. The court should refuse to admit evidence of the relationship only if its probative value is substantially outweighed by the danger of harm and unfair prejudice to the employee. The court should admit evidence of the relationship only if its probative value substantially outweighs the danger of harm and unfair prejudice to the employee. The court should admit evidence of the relationship because the exclusion of evidence offered to prove a victim's sexual conduct or predisposition applies only to a criminal case.

Answer choice C is correct. Evidence offered to prove a victim's sexual conduct or predisposition is generally not admissible in a civil proceeding involving alleged sexual misconduct, such as a sexual harassment action. Such evidence may be admitted, however, when the court determines at an in camera hearing that the probative value of the evidence substantially outweighs the danger of harm to the victim and unfair prejudice to any party. Answer choice A is incorrect because evidence of a victim's sexual conduct is admissible if the court determines that the probative value substantially outweighs the danger of harm and unfair prejudice. Answer choice B is incorrect because it misstates the standard. Such evidence is generally not admissible unless the probative value substantially outweighs the danger of harm and unfair prejudice. Answer choice D is incorrect because such evidence is generally not admissible in either civil or criminal cases.

Question 6484 In a negligence action against an innkeeper, a maintenance man testified for the plaintiff and said that the innkeeper's inn had received poor safety ratings in its last inspection. The innkeeper's attorney later called the maintenance man's co-worker, who testified that the maintenance man had a reputation at work for dishonesty. On his cross-examination of the co-worker, the plaintiff's attorney asked, "Isn't it true that you lied about your qualifications on your résumé?" The innkeeper's attorney objected. Is this a proper question for the plaintiff's attorney to ask on cross examination of the co-worker? Answers: No, because it is not a proper impeachment use of a specific instance of conduct. No, because the co-worker testified only to the maintenance man's reputation. Yes, because it is a proper impeachment question regarding the co-worker's credibility. Yes, because it reveals the co-worker's capacity to comment on the maintenance man's reputation.

Answer choice C is correct. Generally, a specific instance of conduct (e.g., lying on a job application) is not admissible to attack or support the witness's credibility. However, on cross-examination, a witness may be asked about specific instances of conduct if it is probative of the truthfulness or untruthfulness of (i) the witness or (ii) another witness about whose character the witness being cross-examined has testified. Because the cross-examination here is probative of the co-worker's own character for untruthfulness, the question is proper. Answer choice A is incorrect. This is a proper use of a specific instance of conduct because it bears on the co-worker's credibility. Answer choice B is incorrect because the co-worker does not need to open the door for this evidence by discussing his own character. It is sufficient that he is being cross-examined and the event is probative of his truthfulness. Answer choice D is incorrect because the specific-conduct evidence is not admissible for establishing a witness's ability to comment on another person's reputation for truthfulness. This is beyond the scope for which specific-conduct evidence is admissible.

uestion 6487 In a diversity action brought in federal court, a plaintiff seeks to establish that her husband, who has been absent for eight years, is dead, in order to establish her claim to the proceeds from his life insurance policy. State law in the jurisdiction where the court is located provides that an irrebuttable presumption arises that a person is dead when a party establishes that the person has been missing and not heard from for more than seven years. By what law should the court determine whether this presumption should apply? Answers: Federal common law, because federal procedural law controls. Federal statutory law, because federal procedural law controls. The state law, because it is determinative of the existence of a claim or defense. The state law, but the presumption will be considered rebuttable in this federal court action.

Answer choice C is correct. In a federal diversity action, the federal court generally applies the Federal Rules to determine the resolution of evidentiary issues. However, when state substantive law is determinative of the existence of a claim or defense under the Erie doctrine, then state law, rather than the Federal Rules, also governs the effect of a presumption related to the claim or defense. Therefore, because the presumption will affect the plaintiff's claim, the state law presumption controls. Answer choices A and B are incorrect because neither federal common law nor statutory law will control here because the state's rule is determinative of the plaintiff's claim. Answer choice D is incorrect because the conclusive or rebuttable nature of the presumption does not affect its application in a federal diversity case.

Question 621 The owner of an electronics store brought a civil suit for the value of stolen electronics against one of his former employees, who had previously been convicted in a criminal court for the theft of the same goods. During the civil trial, the plaintiff-owner called a witness whom he hoped would testify that she saw the defendant in possession of the stolen goods the day after the electronics store was robbed. The witness, however, testified that she did not see the defendant in possession of the goods, and that she was actually out of town the day after the robbery. The plaintiff seeks to introduce the witness's testimony in the criminal case, in which she testified that she saw the defendant in possession of the goods the next day. The defendant objects to the introduction of the statement. Should the court allow the testimony into evidence? Answers: Yes, for impeachment only. Yes, as substantive evidence only. Yes, for both impeachment and as substantive evidence. No, not for any reason.

Answer choice C is correct. Prior inconsistent testimony is admissible as both impeachment evidence and as substantive evidence. A prior inconsistent statement made under oath at a trial, hearing, or deposition is considered nonhearsay, and is admissible to impeach the declarant's credibility and as substantive evidence. The witness's credibility may be called into question by showing that the witness has previously made a statement that is materially inconsistent with some part of her current testimony. Further, when that statement was made under oath, it may also be used to prove the truth of the matter asserted—in this case, that the defendant was in possession of the goods on the day after the robbery. Because answer choice C is the only choice that reflects that the testimony may be used for both purposes, answer choices A, B, and D are incorrect.

Question 6485 An animal rights activist is on trial for the burglary of a pharmaceuticals lab. The prosecution's theory of the case is that the activist broke into the lab using a maintenance access tunnel connecting the lab to a neighboring building. The tunnel was difficult to locate and was not depicted on any blueprints of the building. The prosecution hopes to call the architect of the lab to testify that the activist visited his office on a number of occasions to interview him for a story, and that the activist had ample time to inspect a three-dimensional model of the planned lab that clearly depicted the access tunnel. The model has since been recycled as part of the architect's regular business practices and cannot be retrieved to be entered as evidence. The defense had no advance notice of the architect's testimony, and objects to the architect's description of the model. Can the architect properly testify as to the appearance of the model? Answers: No, because the prosecution did not establish that no photograph or other reproduction of the model exists. No, because the prosecution did not give advance notice of its intent to introduce this oral testimony. Yes, because the best evidence rule does not apply. Yes, because the model was not destroyed in bad faith.

Answer choice C is correct. The best evidence rule (also known as the original document rule) requires that the original document (or a reliable duplicate) be produced in order to prove the contents of a writing, recording, or photograph, including electronic documents, x-rays, and videos. A "writing" is defined as "letters, words, numbers, or their equivalent set down in any form." A "recording" and "photograph" are similarly broadly defined. However, even under a broad interpretation of these terms, real physical evidence, such as a three-dimensional model, is not subject to the best evidence rule, even though the prosecution in this case contends that the defendant's knowledge of the existence of the tunnel was based on the model. Answer choice A is incorrect. The best evidence rule does not require a party to present the most persuasive evidence, nor does it require the presentation of documentary evidence instead of a witness's testimony simply because a document is available. Therefore, the prosecution does not need to establish that no adequate substitute evidence exists before admitting this oral testimony. Answer choice B is incorrect because there is no requirement for advance notice in this situation, even if the best evidence rule were applicable. Answer choice D is incorrect. Although the intentional destruction of evidence raises a presumption that such evidence would have been unfavorable to the party responsible for destroying it, otherwise inadmissible evidence is not admissible just because it was not destroyed in bad faith. Bad-faith destruction of the evidence would only prevent the architect from testifying about the model if the best evidence rule applied. However, as explained with regard to answer choice C, the best evidence rule is not applicable here.

Question 6965 A woman sued her ex-husband for battery after he allegedly pushed her down the stairs in their marital home following a disagreement. The ex-husband claimed that the woman tripped on a rug and accidentally fell down the stairs. A few days before trial, the ex-husband, who had a serious heart condition, suffered a fatal heart attack. As he was dying, he told a paramedic, "I pushed my ex-wife down the stairs. Please tell her I am so sorry." Which of the following hearsay exceptions will allow admission of the paramedic's testimony as to the ex-husband's statement? Answers: Dying declaration Excited utterance Statement against interest Statement of then-existing state of mind

Answer choice C is correct. The ex-husband's statement is hearsay because it is an out-of-court statement being offered to prove the truth of the matter asserted—that the ex-husband pushed his ex-wife down the stairs. A statement made by a declarant who is unavailable to testify is not excluded as hearsay if the statement was against the declarant's interest at the time it was made, and would not have been made by a reasonable person unless he believed it to be true. Here, the ex-husband is unavailable to testify because he died before the trial. His statement that he pushed his ex-wife down the stairs was against his interest when he made it to the paramedic, and a reasonable person would not make the statement unless he believe it to be true. Answer choice A is incorrect. A statement qualifies as a "dying declaration" if the declarant believes that her death is imminent, and the statement pertains to the cause or circumstances of the death she believes to be imminent. Here, the husband's statement does not pertain to the cause of his heart attack and his subsequent death. Therefore, this hearsay exception does not apply. Answer choice B is incorrect. A statement made about a startling event or condition while the declarant is under the stress of excitement that it caused is not excluded as hearsay. However, because the ex-husband's statement does not relate to a startling event or condition, it does not apply. Answer choice D is incorrect. A statement of present intent, motive, or plan can be used to prove conduct in conformity with that state of mind. Here, the statement does not relate to a present intent, motive, or plan, so this hearsay exception does not apply.

Question 6981 A defendant is on trial for violating a protection order. The defendant had previously been ordered by a court to stay at least 100 feet away from his ex-wife's residence and to refrain from contacting her in any way. One morning, while the ex-wife was out shopping, the defendant visited her home. The ex-wife's sister, who lived with the ex-wife, answered the door and told the defendant that the wife was not there. The defendant left without further incident. When the wife returned home two hours later, her sister told her, "That no-good ex-husband of yours was here a few hours ago. Isn't he supposed to stay away from the house?" At trial, the prosecution wants to call the ex-wife to the stand to testify about her sister's statement. Is the ex-wife's testimony admissible? Answers: Yes, because the sister's statement is a present sense impression. Yes, because the sister's statement is an excited utterance. No, because the sister's statement is inadmissible hearsay. No, because the sister has not been shown to be unavailable.

Answer choice C is correct. The sister's statement is an out-of-court statement offered for the truth of the matter asserted and is thus hearsay. The statement does not fall within any hearsay exception. Therefore, the ex-wife's testimony regarding the sister's statement is inadmissible. Answer choice A is incorrect because the sister did not make her statement while or immediately after perceiving the defendant's visit to the ex-wife's home. Answer choice B is incorrect. A statement made about a startling event or condition while the declarant is under the stress of excitement that it caused is not excluded as hearsay. Under this exception to the hearsay rule, the event must shock or excite the declarant, and the statement must relate to the event, but the declarant need not be a participant in the event. In this case, the sister did not make the statement to the ex-wife until two hours after the defendant left the home without incident. Even if she was still angry or annoyed at the defendant for the visit, it is very unlikely that she was still under the stress of excitement that the visit caused two hours later. Answer choice D is incorrect. There are five exceptions to the hearsay rule that apply only if the declarant is unavailable as a witness: former testimony, dying declaration, statement against interest, statement of personal or family history, and statement offered against a party that wrongfully caused the declarant's unavailability. Because none of these exceptions apply to this situation, the statement would be inadmissible even if the sister had been shown to be unavailable.

Question 1966 A defendant was charged with possession of marijuana with intent to distribute. On direct examination, the defendant testified that he worked with disadvantaged children as a drug counselor, that he hated drugs, that he would "never possess or distribute drugs," and that he had never used drugs and would not touch them. The government offered as a rebuttal witness a police officer who would testify that, three years earlier, he saw the defendant buy cocaine from a street dealer. The defendant objected. Is the testimony of the police officer about the prior drug transaction admissible to impeach the defendant? Answers: No, because the bad act of buying drugs is not sufficiently probative of a witness's character for truthfulness. No, because it is contradiction on a collateral matter. Yes, because it is proper contradiction. Yes, because the bad act shows a disregard for the law and makes it less likely that the defendant would respect the oath of truthfulness.

Answer choice C is correct. This is an example of impeachment by contradiction. The evidence of the prior cocaine purchase directly contradicts the defendant's testimony on direct examination that he would never possess drugs and is admissible for that purpose under 403. Answer choice A is incorrect. The evidence of purchasing cocaine is not being offered to impeach the defendant's character for truthfulness. Instead it is being offered to contradict his testimony on direct examination that he would never possess drugs. The offered testimony is admissible for that purpose. Answer choice B is incorrect. The evidence of a prior cocaine purchase is not collateral, because the defendant is charged with drug possession and the evidence directly contradicts the defendant's testimony on direct examination that he would never possess drugs. The offered testimony is admissible for that purpose. Answer choice D is incorrect. The evidence of the prior cocaine purchase is not being offered under Federal Rule of Evidence 608(b) to show a specific instance of conduct bearing on the witness's truthfulness. The evidence would not be admissible under Rule 608(b) in any case, because it prohibits extrinsic evidence of specific instances of conduct and only allows questioning about them on cross-examination. Instead, this is an example of impeachment by contradiction, which is governed by 403. The evidence of the prior cocaine purchase directly contradicts the defendant's testimony on direct examination that he would never possess drugs and is admissible for that purpose.

Question 6260 A plaintiff hopes to introduce an original x-ray image of his broken arm to help prove the injury element of his negligence claim against the defendant. The plaintiff called a doctor to lay the foundation in order to enter the image into evidence. At trial, the doctor testified that the image was an accurate reproduction of the plaintiff's arm taken the day after the alleged injury. The doctor also testified that he had taken the x-ray image himself. If the above testimony is the only testimony presented about the x-ray image, is the image admissible? Answers: Yes, because the doctor authenticated the image. Yes, because the image is self-authenticating. No, because the image has not been properly authenticated. No, because the best evidence rule bars the image's admission.

Answer choice C is correct. X-ray images, electrocardiograms, and similar items are physical representations of things that cannot otherwise be seen (i.e., the inner workings and functionality of a human body), and, as such, unlike other reproductions, they cannot simply be authenticated by the testimony of a witness claiming that they are accurate reproductions of the facts. To authenticate such an item, it must be shown that an accurate process was used, that the machine used was working properly, and that the operator of the machine was qualified to operate it. The chain of custody must also be established. These facts have not been established here. As a result, the image has not been properly authenticated and thus is not admissible. Answer choice A is incorrect because the doctor's testimony is insufficient to authenticate an x-ray image. Answer choice B is incorrect because an x-ray image is not self-authenticating. Answer choice D is incorrect because the best evidence rule only requires that the original document be produced in order to prove the contents of an x-ray image, and the image being introduced here is the original.

Question 6270 A teller is on trial for the crime of obstructing justice during a Securities and Exchange Commission (SEC) investigation. The teller is accused of intentionally hiding key financial documents after the investigation was announced. The government calls the bank's attorney to testify that when the SEC announced its investigation, the teller asked him how to comply with the regulations, what financial documents needed to be released, and what the potential consequences would be for failing to provide the required documents. The bank's attorney, unaware of the teller's plans to conceal these documents, said that he would look into it and let the teller know the next day. The teller objects, claiming the attorney-client privilege. Should the court admit the bank attorney's testimony? Answers: No, because the attorney did not know that the teller had an illegal purpose. No, because the teller is not in a position to control the bank's corporate decisions. Yes, because the attorney did not give the teller legal advice. Yes, because the teller knew or should have known that he intended to commit a crime or fraud.

Answer choice D is correct. A confidential communication between a client and an attorney for the purpose of seeking legal advice or representation is privileged. However, the attorney-client privilege does not protect communications made to enable or aid in the commission of what the client knew or should have known was a crime or fraud. Answer choice A is incorrect because the attorney's knowledge of the client's criminal purpose is not necessary to make the communication admissible. It is the client's awareness of the criminal nature of his actions that defeats the privilege. Answer choice B is incorrect because the attorney-client privilege extends to communications with a corporate lawyer by a non-control-group employee about matters within the employee's corporate duties made for the purpose of securing legal advice for the corporation. Answer choice C is incorrect because the attorney does not need to give advice or agree to the representation for the privilege to exist.

Question 6265 A plaintiff brought an action for negligence arising out of a car accident against the defendant. The plaintiff contends that the defendant drove through a red light, colliding with the plaintiff's car. The defendant's wife was a passenger in the defendant's car. The plaintiff's attorney wants to call a bystander, who was present at the scene of the accident, to testify that he heard the defendant's wife loudly exclaim immediately after the collision, "The light was red! I can't believe you didn't stop!" Is the bystander's testimony admissible? Answers: No, because it contains a privileged spousal communication between the defendant and his wife. No, unless the defendant's wife fails to object to the bystander's testimony. Yes, because it is non-hearsay. Yes, because it is an excited utterance.

Answer choice D is correct. A statement made about a startling event or condition while the declarant is under the stress of excitement caused by the event is admissible as an excited utterance. The event must shock or excite the declarant, and the statement must relate to the event, but the declarant need not be a participant in the event (i.e., can be a bystander). The defendant's wife's statement falls within this exception to the hearsay rule. Therefore, the bystander can testify as to her statement. Answer choice A is incorrect. A communication made between spouses while they are married is privileged if the communication was made in reliance upon the sanctity of marriage. However, the wife's statement was made so loudly that it cannot be considered to have been made in reliance upon the sanctity of the marriage (i.e., not confidential). Answer choice B is incorrect because spousal privilege does not apply. Answer choice C is incorrect because the testimony is hearsay but it falls within the excited utterance exception. The wife's statement does not qualify as an opposing party's statement because the husband, not the wife, is the defendant in this case.

Question 7501 A defendant was charged with involuntary manslaughter. The defendant testified that he did not shove the victim, but that instead she had tripped and fell down the stairs. Subsequently, the prosecution called a police officer who had talked with the defendant at the scene of the incident. The police officer testified that, on the day of the incident, the defendant told the officer that he had shoved the victim before she fell down the stairs. The prosecution had not asked the defendant about this statement when he was on the witness stand. Is this statement admissible as substantive evidence of the defendant's actions? Answers: No, because the prosecution did not ask the defendant about the statement when the defendant was on the witness stand. No, because the statement was not made under oath. Yes, because it contradicts the defendant's testimony. Yes, because the defendant made the statement to the police officer.

Answer choice D is correct. A statement made by a party to the current litigation is not hearsay if it is offered by an opposing party, even though the statement is being offered to prove the truth of the matter asserted. Consequently, the defendant's statement to the police officer that the defendant shoved the victim before she fell down the stairs is admissible, even though if made by someone else, it would be inadmissible hearsay. Answer choice A is incorrect because a party need not be confronted with a prior statement made by the party before the opposing party can introduce the statement at trial as substantive evidence. Answer choice B is incorrect. Although a prior inconsistent statement made by a witness generally must have been made under oath in order to be admitted as substantive evidence, this is not the case with regard to an opposing party's statement. Answer choice C is incorrect because an opposing party's statement is admissible as substantive evidence, regardless of whether the opposing party has even testified or whether the statement contradicts that testimony. (Note: This statement would not be admissible as a declaration against interest because the defendant, having testified, is not unavailable.)

Question 6259 A plaintiff sued a company for damages incurred when the company's delivery truck drove through the plaintiff's storefront. The plaintiff wants to testify that the driver, a permanent employee of the company, told the plaintiff, "I kept telling my boss these brakes were on their last miles, but they were not fixed. They failed as I turned that corner." The company owns the truck. Is the plaintiff's testimony regarding the truck driver's statements likely admissible? Answers: No, because the testimony is hearsay not within any exception. No, because there is no proof that the truck driver was authorized to speak for the company. Yes, the testimony is admissible under a hearsay exception. Yes, the testimony is admissible as non-hearsay.

Answer choice D is correct. A statement made by a party to the current litigation is not hearsay if it is offered by an opposing party. A statement made by a party's agent or employee constitutes an opposing party's statement if it was made concerning a matter within the scope of and during the course of the relationship (i.e., a vicarious admission). Here, the truck driver's statement is a vicarious admission because it is a statement (i) made by an agent or employee of the company during the course of the employment relationship and (ii) it concerned a matter within the scope of the truck driver's employment. Therefore, the truck driver's statement is admissible as non-hearsay. Answer choices A and C are incorrect because a vicarious admission is not hearsay if it is offered by an opposing party. Answer choice B is incorrect because there is no requirement that the truck driver be authorized to speak on behalf of the company as long as the truck driver was an agent or employee of the company, speaking about a matter within the scope of his employment during the course of the employment relationship.

uestion 5847 In a civil trial for aggravated battery, the defendant testified that he was aiming at a squirrel and did not see the plaintiff when he fired his gun. The plaintiff did not cross-examine the defendant regarding this issue. The plaintiff then called a witness who testified that a few days after the alleged aggravated battery, the defendant said to the witness, "As soon as I saw the plaintiff, I fired my gun right at him." The defendant objected to the witness's testimony. Is the witness's testimony admissible? Answers: No, because the defendant was not first given an opportunity to explain or deny the statement. No, because it is hearsay that does not fall within an exception. Yes, but only to impeach the defendant. Yes, both as evidence that the defendant committed battery and to impeach the defendant.

Answer choice D is correct. A statement made by a party to the current litigation is not hearsay if it is offered by an opposing party. In this case, the defendant's statement to the witness was offered by the plaintiff. Thus, the statement is an opposing party's statement and not hearsay. It may be used both as substantive evidence to prove the defendant committed battery and to impeach the defendant. Answer choice A is incorrect. Extrinsic evidence (i.e., evidence other than the witness's own testimony) of a witness's prior inconsistent statement may be introduced only if the witness is given the opportunity to explain or deny the statement, and the opposing party is given the opportunity to examine the witness about it. The witness's opportunity to explain or deny the statement need not take place before the statement is admitted into evidence. The opportunity to explain or deny a prior inconsistent statement does not apply when the statement qualifies as an opposing party's statement. Here, the defendant's statement to the witness is an opposing party's statement. Therefore, the defendant need not be given the opportunity to explain or deny the statement. Answer choice B is incorrect because the defendant's statement is an opposing party's statement and therefore not hearsay. Answer choice C is incorrect because the witness's testimony may also be used as substantive evidence because it is an opposing party's statement.

Question 3017 A man and a woman were each charged with first degree murder and conspiracy to commit murder. The woman and her attorney entered into plea discussions with the prosecution. The woman told the prosecutor that the man had come up with the idea to murder the victim and had fired the shot that killed the victim. She agreed to plead guilty to a lesser charge in exchange for her testimony against the man. The woman attended a hearing on the record with her counsel and pleaded guilty. The woman later moved to withdraw her guilty plea and proceed to trial. Because the judge had neglected to notify the woman of her right to a jury trial at the plea hearing, he granted her motion and the woman proceeded to trial. The prosecution intends to introduce the woman's statements during the plea negotiations, as well as the fact that she previously entered a guilty plea. Which of the following is admissible against the woman at trial? Answers: Both the guilty plea and the statements during the plea negotiations. The guilty plea only. The statements during the plea negotiations only. Neither the guilty plea nor the statements during the plea negotiations.

Answer choice D is correct. A statement made during a negotiation between a prosecutor and a defendant regarding a plea that does not result in a plea is not admissible. A withdrawn guilty plea is likewise inadmissible. Answer choices A, B, and C are incorrect because neither statements made during plea negotiations nor withdrawn guilty pleas are admissible under Rule 410.

Question 4206 A man witnessed a hit-and-run accident in which a truck struck and killed a child. Shortly after the accident, the witness gave police a signed, handwritten statement with a description of the truck, including the make and model, as well as a description of the driver of the truck. After several months, the authorities identified and charged a man who fit the witness's description, although the pickup truck was never located. In the interim, however, the witness had suffered a brain injury and no longer remembered any of the events from the date of the accident. At trial, the prosecutor called the witness to testify. The prosecutor first showed the witness a photo of a vehicle of the same make and model as the truck in an attempt to refresh the witness's memory. The defense objected but was overruled by the judge. When this failed to refresh the witness's memory, the prosecutor showed the witness his earlier handwritten and signed statement. Again, the defense objected and was overruled. The witness testified that he had no memory of the events described in the statement, but that he recognized his handwriting on the statement and that the statement accurately reflected what he witnessed at the time. The prosecutor then moved to introduce the photo and statement into evidence as exhibits, and the defense again objected. How should the judge rule? Answers: The judge should admit both the photo and the statement into evidence as exhibits. The judge should admit the photo, but not the statement, into evidence as an exhibit. The judge should admit the statement, but not the photo, into evidence as an exhibit. The judge should not admit the photo or the statement into evidence as exhibits.

Answer choice D is correct. A witness may examine any item (e.g., writing, photograph) in order to refresh the witness's present recollection, but the witness's testimony must be based on the witness's refreshed recollection, not on the item itself. When the item used to refresh a witness's recollection is a writing or other record, the adverse party is entitled to have the document produced, to inspect the document, to cross-examine the witness about it, and to introduce any relevant portion into evidence. If a witness is unable to testify about a matter for which a record exists, that record will satisfy the recorded recollection exception to the hearsay rule if: the record is on a matter that the witness once knew about; the record was made or adopted by the witness when the matter was fresh in the witness's memory; the record accurately reflects the witness's knowledge; and the witness states that he cannot recall the event well enough to testify fully and accurately, even after consulting the record on the stand. If a record is admitted under this exception, it may be read into evidence, but it may be received as an exhibit only if offered by an adverse party. Here, the prosecutor presented the photos to the witness in an attempt to refresh the witness's memory and offered the witness's statement as a recorded recollection. Only the defense would be able to offer the record into evidence. The photo is not admissible because the witness neither authenticated it nor established its relevance. Although the witness's statement qualifies as a recorded recollection, only the adverse party (here, the defense) may offer it as an exhibit into evidence. Thus, the judge should not admit it based on the prosecutor's motion. Answer choice A is incorrect because neither the photo nor the statement is admissible as an exhibit. Answer choice B is incorrect because the witness neither authenticated nor established the relevance of the photo. Answer choice C is incorrect because only the adverse party (here, the defense) may offer the statement as an exhibit into evidence.

Question 4203 A car struck a truck at an intersection. The driver of the truck sued the driver of the car, claiming that the car driver ran a red light. In the investigating officer's report, the only witness is quoted saying, "I saw the whole thing. The car had the green light." At trial, the witness testified that he clearly remembered that the car's traffic light had been red, and that the car ran the light. The defendant did not cross-examine the witness and the witness was dismissed and left the jurisdiction. After the plaintiff had presented his case, the defendant moved to introduce the witness's statement from the investigating officer's report solely to impeach the witness's testimony. The plaintiff objected. How should the court rule? Answers: Overrule the objection, and admit the statement as substantive evidence that the car driver did not run the red light. Overrule the objection, and admit the statement as impeachment evidence only. Sustain the objection, because the statement is inadmissible hearsay. Sustain the objection, because extrinsic evidence may not be used to impeach a witness under the circumstances.

Answer choice D is correct. A witness's prior statement that is inconsistent with a material part of the witness's testimony may be used to impeach the witness. However, extrinsic evidence of a witness's prior inconsistent statement may be introduced only if the witness is given the opportunity to explain or deny the statement and the opposing party is given the opportunity to examine the witness about it. In this case, because the witness was not given an opportunity to explain or deny the statement, the evidence should be excluded. Answer choice A is incorrect because, although the statement is relevant evidence on the substantive issue of whether the car driver did run the red light, the defendant has sought to use the statement only for impeachment purposes (i.e., to call into question the veracity of the witness's testimony at trial). Answer choice B is incorrect because extrinsic evidence is not admissible for impeachment purposes unless the witness is given the opportunity to explain or deny it. Answer choice C is incorrect because, while the statement would be inadmissible hearsay if it were introduced for its truth, the statement is being introduced for impeachment purposes only.

Question 6979 A plaintiff and a defendant were involved in a car accident in the middle of an intersection. A police officer arrived, and after the officer took down all the basic information of both drivers and spoke with eyewitnesses and the two parties, the defendant said to the officer, "I ran a red light." The officer recorded this statement in his police report. The defendant also said to the plaintiff, "Don't worry, I have car insurance." During a trial for negligence, the plaintiff attempts to admit both statements against the defendant. Which of the defendant's statements are admissible? Answers: Both statements Neither statement Only the statement regarding car insurance Only the statement regarding running the red light

Answer choice D is correct. Although a police report can qualify under the business records exception, a statement made by a witness that is contained in the report does not generally qualify because the witness is not acting on behalf of the police in making the statement. The statement may, however, qualify as an opposing party's statement, which is a statement made by a party to the current litigation and is not hearsay if it is offered by an opposing party. Here, the statement in the police report would qualify as an opposing party's statement as it was made by the defendant. For this reason, answer choices B and C are incorrect. Answer choice A is incorrect. Evidence that a person was or was not insured against liability is not admissible to prove whether the person acted negligently or otherwise wrongfully. Here, the defendant told the plaintiff not to worry because he had car insurance. Accordingly, the plaintiff is not permitted to admit this statement to prove that the defendant acted negligently. Because there is no other reason for admitting this statement, it is inadmissible under a public policy exclusion.

Question 6258 An officer stepped out of a coffee shop to find a woman standing on the sidewalk, visibly shaken and clutching a purse with a torn handle. She looked at the officer and stammered, "He just tried to take it, that hooligan in the red jacket! He had a gun!" She pointed down the street in the direction that the hooligan had run. "He just ran around that corner." The officer jumped in his car and described the incident to another officer over the radio. The second officer replied, "I may have just seen him running down Main, so you could probably catch him on Oak Street." The officer drove to Oak Street and arrested an innocent man who was running and also wearing a red jacket. The man is now suing the officer for false arrest. The man has filed motions to exclude both the statements of the woman and the statements of the second officer. If the court determines that these statements are hearsay, how should the court rule on these motions? Answers: Grant both motions. Grant only the motion regarding the second officer's statement over the radio. Grant only the motion regarding the woman's description of the hooligan. Deny both motions.

Answer choice D is correct. Although both statements are hearsay, they fall within the "present sense impression" exception. A statement describing or explaining an event that is made while the declarant is perceiving the event, or immediately thereafter, is admissible under the present sense impression exception to the hearsay rule, regardless of whether the declarant is available to testify. Although the man may argue that the woman's testimony is a testimonial statement, this is not a strong argument; testimonial statements include statements made during a police interrogation that had the primary purpose of ascertaining past criminal conduct. A statement made to police during the course of questioning with the primary purpose of enabling police to provide assistance to meet an ongoing emergency, such as a person informing an officer of an attempted armed robbery that just occurred and telling the officer where the armed robber went, is not testimonial. (Note: The woman's statement would likely also fall under the "excited utterance" exception to hearsay). Accordingly, answer choices A, B and C are incorrect because all of the statements are admissible.

Question 3054 A handyman sued a homeowner for negligence after falling off of the homeowner's roof while engaged in repairing the homeowner's chimney. The handyman was knocked off of the roof when one of several large dead branches from a nearby tree fell on him. The homeowner's answer contested that the tree was on the homeowner's property. At trial, counsel for the handyman seeks to introduce into evidence a properly authenticated invoice, made in the regular course of business, from a landscaping company to the homeowner that indicated that the tree's dead branches were subsequently trimmed at the homeowner's expense. The homeowner objected to the admission of the invoice. Should the judge admit the invoice into evidence over the homeowner's objection? Answers: No, because it is hearsay not within any exception. No, because it evidences a subsequent remedial measure. Yes, because it is a vicarious admission by the homeowner. Yes, because ownership of the tree was disputed.

Answer choice D is correct. Although evidence of subsequent remedial measures is generally not admissible to prove negligence for public policy reasons, it may be admissible for other purposes. One such purpose is to prove ownership or control of the cause of harm, if disputed. Because ownership was disputed here, it is proper for the plaintiff to introduce evidence tending to prove that the defendant owned the tree. Accordingly, answer choice B is incorrect. Answer choice A is incorrect because the bill would constitute a record of a regularly conducted activity and thus fall under the business records exception to the hearsay rule. Answer choice C is incorrect because the existence of an invoice that the defendant purportedly paid would not constitute a vicarious admission by the homeowner. A vicarious statement is a statement made by one person but imputed to another based on their relationship. Not only would a landscaper not qualify as an employee or agent of a homeowner for these purposes, it is the very act of payment, not any statements by the landscaper and attributed to the homeowner, that are at issue.

Question 4327 At a street fair, a clown in a dunking booth shouted insults to anyone who walked by. Unless the recipient of the clown's comments stopped to try their hand at dunking him, most simply rolled their eyes or walked by without comment. When one woman walked by, the clown said, "I know you killed your husband. Folks, doesn't she look like a husband killer?" The woman stared at the clown for a second, as did a group of onlookers, but said nothing and kept walking by. As the woman turned a corner, she began to panic; she really had just killed her husband, but had no idea how the clown could have known that. When she reached the end of the block, she turned around, jogged back, and yelled to the clown, "No, I didn't!" Did the woman adopt the clown's statement when she remained silent? Answers: Yes, because she did not respond to the clown right away. Yes, because she had just killed her husband. No, because she eventually responded to the clown's comment. No, because a reasonable person would not have denied the statement.

Answer choice D is correct. An adoptive admission is a statement of another person that a party expressly or impliedly adopts as his own. Silence in response to a statement is considered an adoptive admission if: (i) The person was present and heard and understood the statement; (ii) the person had the ability and opportunity to deny the statement; and (iii) a reasonable person similarly situated would have denied the statement. Here, the woman was present and had the opportunity to deny the statement; a similarly situated reasonable person, however, would not have felt compelled to answer a seemingly random accusation by a clown hurling insults at a street fair. Answer choice A is incorrect because a reasonable person would not have felt the need to respond to an unsubstantiated and inflammatory accusation shouted at them by a clown at a street fair. Answer choice B is incorrect because whether the statement was true or not is irrelevant to whether it was adopted. The statement would still not be an adoptive admission if a reasonable person would not have felt compelled to respond. Answer choice C is incorrect because the woman's later denial did not affect whether she had made an adoptive admission at the time the statement was made.

Question 6264 An athlete sued a driver for damages for injuries sustained in a car accident. The driver disputes the damages and seeks to prove that the athlete's claimed injuries resulted from years of repetitive concussions during his athletic career. The athlete argues that, although he has sustained a number of concussions over the years, the car accident was the sole cause of his present injuries. To support his argument, the athlete called a physician as an expert witness. The athlete's lawyer asked the physician a question regarding the possible causes of the athlete's particular injury by posing a hypothetical scenario that omitted any reference to the effects of repetitive concussions. The physician had never examined the athlete, and the question was not provided to opposing counsel before trial. Which of the following is the driver's best ground upon which to object to this question? Answers: Hypothetical questions are not permitted when they embrace an ultimate issue in the case. Opposing counsel did not give sufficient notice of the hypothetical question before trial. The physician had no personal knowledge because he did not examine the athlete. The hypothetical question omitted a clearly significant fact rendering the testimony not relevant.

Answer choice D is correct. Before an expert witness may testify, the court must first determine whether the subject matter of the witness's testimony (i) is scientific, technical, or otherwise specialized knowledge, which focuses on the reliability of the testimony; and (ii) will help the trier of fact understand the evidence or determine a fact in issue, which focuses on the relevance of the testimony. Here, a hypothetical question that omits a key fact affecting the issue before the court is not helpful to the jury. Therefore, the best argument is that the hypothetical is not relevant. Answer choice A is incorrect. Although an expert may not state an opinion about whether a criminal defendant has the requisite mental state of any element of the crime charged or of a defense, an expert's opinion may be admissible even though the opinion embraces an ultimate issue in the case. Answer choice B is incorrect because no such notice is required by law. Answer choice C is incorrect because personal knowledge is not required. An expert may base her opinion on personal observation, facts made known to the expert at trial, or facts not known to the expert personally but supplied to her outside the courtroom, if they are the type reasonably relied upon by experts in the field.

Question 5918 A plaintiff was injured while touring a clothing factory. On the date of the accident, a factory employee, who had witnessed the accident, wrote down the name of the plaintiff, the exact circumstances of the accident, and the time and date. Although the factory did not have a policy of keeping accident reports, the factory employee thought that doing so might be useful in case the plaintiff decided to file a lawsuit. Soon after the accident, the plaintiff brought an action against the factory owner for negligence. At trial, the factory owner called the factory employee to testify as to the accident. Although he testified that the written report accurately reflected his knowledge when it was made, the factory employee was unable to sufficiently recall the accident, even after reviewing the report. The factory owner then sought to read the written report into evidence. May the factory owner do so? Answers: No, because it constitutes hearsay not within an exception. No, because it may only be offered into evidence by the plaintiff. Yes, under the business records exception to the hearsay rule. Yes, under the past recollection recorded exception to the hearsay rule.

Answer choice D is correct. If a witness is unable to testify about a matter for which a record exists, that record is not excluded as hearsay if (i) the record is on a matter that the witness once knew about, (ii) the record was made or adopted by the witness when the matter was fresh in the witness's memory, (iii) the record accurately reflects the witness's knowledge, and (iv) the witness states that she cannot recall the event well enough to testify fully and accurately, even after consulting the record on the stand. In this case, the factory employee witnessed the accident and wrote down his observations and other relevant information on the date of the accident. The factory employee testified that the written report accurately reflected his knowledge when it was made and that he could not sufficiently recall the accident, even after consulting the report. Thus, all of the requirements for the past recollection recorded exception to the hearsay rule have been met. As a result, the written report is admissible. Answer choice A is incorrect because the written report qualifies under the past recollection recorded exception to the hearsay rule. Answer choice B is incorrect because the written report may be read into evidence by either party, though it may only be admitted as an exhibit by the adverse party. Answer choice C is incorrect. Records prepared in anticipation of litigation, such as an employee's accident report, may not qualify under the business records exception due to a lack of trustworthiness. In this case, the facts indicate that the factory employee created the written report because it might be useful in case the plaintiff filed suit. Thus, it was likely created in anticipation of future litigation. Furthermore, the making and keeping of written accident reports was not a regularly conducted activity or practice of the factory. As a result, the business records exception to hearsay does not apply.

Question 6958 In a medical malpractice case, a patient sued her surgeon for allegedly causing organ damage during a routine surgery. In a previous case, the patient sued her medical insurance company for not covering some of the medical expenses associated with the complications from the organ damage. During that trial, the patient testified that she overheard a nurse assisting the surgeon tell another assisting nurse during the surgery that it appeared the patient's kidney had been pierced. Before the patient could testify about this statement in the medical malpractice trial, she died from complications stemming from the surgery. Is the patient's testimony from the previous trial admissible in the medical malpractice trial? Answers: Yes, because the patient is not available and the testimony was given as a witness at trial. Yes, because the patient is not available and the surgeon caused the patient's unavailability. No, because the testimony was given during a previous trial, not the current proceeding. No, because the defense did not have an opportunity to question the patient in the previous trial.

Answer choice D is correct. Testimony that was given as a witness at a trial, hearing, or lawful deposition is not excluded as hearsay if the party against whom the testimony is being offered (or, in a civil case, a party's predecessor-in-interest) had an opportunity and similar motive to develop the testimony by direct examination, redirect examination, or cross-examination. Here, the patient died before she had a chance to testify at the malpractice trial and was thus unavailable. However, the defense in the medical malpractice trial did not have the opportunity to examine the patient during the previous trial. Additionally, the insurance company is not the surgeon's predecessor in interest. Therefore, the statement does not fall within the former testimony exception to hearsay and is inadmissible. Answer choice A is incorrect. Although the patient is not available and gave testimony as a witness during the previous trial, there is an additional requirement that the party against whom the testimony is being offered must have had an opportunity and similar motive to develop the testimony by direct examination, redirect examination, or cross-examination. Answer choice B is incorrect. A statement offered against a party that wrongfully caused the declarant's unavailability is not excluded as hearsay. Under this exception, the wrongful party forfeits the right to object to the admission of the declarant's statement as hearsay. The wrongdoing, which need not be criminal, must be done with the intent of preventing the witness from testifying. Here, the surgeon's alleged wrongdoing was not done with the intent of preventing the patient from testifying at the malpractice trial. Accordingly, this exception does not apply. Answer choice C is incorrect. The former testimony exception applies whether the testimony was given during the current proceeding or during a different one, as long as the witness is now unavailable.

Question 6256 In a prosecution of a man for murder, the government seeks to introduce the confession of a co-conspirator as an admission of a party opponent. In laying the foundation for admitting the confession, the prosecution offered an affidavit from the man's son concerning the friendship between the man and the confessing co-conspirator. Who rules on the admissibility of the co-conspirator's confession, and may the son's affidavit be considered? Answers: The jury determines the confession's admissibility as a question of weight and credibility, and the jury must not consider the affidavit. The jury determines the confession's admissibility as a question of weight and credibility, and the jury may properly consider the affidavit. The judge determines the confession's admissibility as a preliminary question of fact, and the judge must not consider the affidavit. The judge determines the confession's admissibility as a preliminary question of fact, and the judge may properly consider the affidavit.

Answer choice D is correct. The judge must first consider whether the confession of the co-conspirator qualifies as an admission of a party opponent, which is not hearsay. This is a preliminary question regarding the competency of the evidence. The judge must rule on the admissibility of the confession before a jury can determine the weight and credibility of the confession. Accordingly, answer choices A and B are incorrect. Answer choice C is incorrect because the court is not bound by the Federal Rules of Evidence in deciding preliminary questions regarding the competency of evidence (except with respect to privileges), and thus the court may consider otherwise inadmissible evidence. Therefore, there is no reason that the judge cannot consider the son's affidavit.


Related study sets

Psych 178 Final LESSS GOOOO **** DIS FOOOOOOOOO

View Set

Session 6: License renewal and insurance requirements

View Set

Chapter 19, Nursing Management: Postoperative Care

View Set